• Shuffle
    Toggle On
    Toggle Off
  • Alphabetize
    Toggle On
    Toggle Off
  • Front First
    Toggle On
    Toggle Off
  • Both Sides
    Toggle On
    Toggle Off
  • Read
    Toggle On
    Toggle Off
Reading...
Front

Card Range To Study

through

image

Play button

image

Play button

image

Progress

1/297

Click to flip

Use LEFT and RIGHT arrow keys to navigate between flashcards;

Use UP and DOWN arrow keys to flip the card;

H to show hint;

A reads text to speech;

297 Cards in this Set

  • Front
  • Back

Investigation of a suspected anaphylactic reaction requires measurement of tryptase
levels. Correct statements regarding tryptase include each of the following EXCEPT
A. 99% of body tryptase is in mast cells
B. a concentration of greater than 20 ng/mL suggests an anaphylactic reaction
C. blood samples should be repeated 24 to 48 hours after the reaction
D. maximum blood concentrations occur within 1 hour of the reaction
E. tryptase concentrations rise after both anaphylactic and anaphylactoid reactions

A - 99% of body tryptase is in mast cells - true
B - concentration >20ng/ml suggests anaphylaxis - true
C - blood samples at 24-48 hours - false - hence the answer as 6-24hrs is best
D - max blood concs within 1 hour - true (perhaps slightly tempered as max conc within 1 hour of degranulation and question says within 1 hour of reaction, but the reaction is due to degranulation of course . . .
E - tryptase up after anaphylaxis and anaphylactoid - true

Which of the following statements regarding anaphylactic
and anaphylactoid reactions is FALSE?
A. cross sensitivity between latex and bananas,
chestnuts and avacado have been reported
B. cross-sensitivity of cephalosporins with penicillin is
about 8%
C. gelatin solutions used for resuscitation can worsen
any reaction
D. reactions to neuromuscular blocking agents are more
common in females
E. vecuronium is more likely to cause an anaphylactoid
rather than an anaphylactic reaction

A. cross sensitivity between latex and bananas, chestnuts and avacado have been reported - true: "There is a recognized cross-reactivity between latex sensitivity and certain foods, especially bananas, chestnuts and avocado." (Anaphylaxis, in Continuing Education in Anaesthesia, Critical Care & Pain 2004 4(4):111-113)
B. cross-sensitivity of cephalosporins with penicillin is about 8% - true: "Penicillins are most frequently implicated in hypersensitivity reactions. The incidence of cross-reactivity with cephalosporins is about 8%." (Anaphylaxis, in Continuing Education in Anaesthesia, Critical Care & Pain 2004 4(4):111-113)
C. gelatin solutions used for resuscitation can worsen any reaction - true: "Fluids used for resuscitation after anaphylaxis may themselves cause histamine release and worsen any reaction. The risk is greatest with gelatin solutions." (Anaphylaxis, in Continuing Education in Anaesthesia, Critical Care & Pain 2004 4(4):111-113)
D. reactions to neuromuscular blocking agents are more common in females - true: "This could explain why anaphylaxis to neuromuscular blocking agents is five to ten times more common in females." (Anaphylaxis, in Continuing Education in Anaesthesia, Critical Care & Pain 2004 4(4):111-113)
E. vecuronium is more likely to cause an anaphylactoid rather than an anaphylactic reaction - false and answer to choose: "Steroid-based compounds (vecuronium and pancuronium) cause anaphylactic reactions, whereas benzylisoquinoliniums (mivacurium and atracurium) tend to cause anaphylactoid reactions." (Anaphylaxis, in Continuing Education in Anaesthesia, Critical Care & Pain 2004 4(4):111-113)

Post-transfusion hepatitis in Australia is associated with

A. jaundice in over 50% of patients
B. development of chronic disease in less than 10% of patients
C. hepatitis B in the majority of patients
D. the presence of antigen or antibody to hepatitis C
E. elevation of serum alkaline phosphatase
C
Which of the following statements is INCORRECT? Recombinant Factor VIIa
A. directly activates Factors IX and X on the surface of activated platelets, leading to thrombin formation
B. has a half-life of three and a half hours
C. has been used "off-label" for bleeding in trauma patients
D. is best monitored by the prothrombin time, which is shortened in a dose-dependent manner at therapeutic doses
E. was developed for patients with haemophilia A and B who have inhibitors to Factors VIII and IX respectively
A. directly activates Factors IX and X on the surface of activated platelets, leading to thrombin formation
B. has a half-life of three and a half hours
C. has been used "off-label" for bleeding in trauma patients - true
D. is best monitored by the prothrombin time, which is shortened in a dose-dependent manner at therapeutic doses - false and answer to choose
E. was developed for patients with haemophilia A and B who have inhibitors to Factors VIII and IX respectively - true
The most frequently reported cause of mortality associated with transfusion of blood and blood products is
A. anaphylaxis
B. bacterial sepsis
C. haemolytic reaction
D. transfusion associated graft versus host disease
E. Transfusion Related Acute Lung Injury (TRALI)
TRALI
In a patient requiring FFP where the patient’s blood group is unknown, it is ideal to give FFP of group
A. A
B. B
C. AB
D. O
E. Blood group of FFP in this situation does not matter
A. A
B. B
C. AB - CORRECT: "Group AB FFP can be used in an emergency if the patient’s ABO blood group is unknown, but is likely to be in short supply... With regard to ABO blood groups, the first choice of FFP is that of the same ABO group as the patient. If this is not available, FFP of a different ABO group is acceptable so long as it has been shown not to possess anti-A or anti-B activity above a limit designed to detect ‘high titres’. FFP of group O should only be given to O recipients" (British Committee for Standards in Haematology, Blood Transfusion Task Force, Guidelines for the use of fresh-frozen plasma, cryoprecipitate and cryosupernatant, in British Journal of Haematology 2004:126, 11–28)
D. O
E. Blood group of FFP in this situation does not matter
A young woman with type 1 von Willebrand disease presents for a dilatation and curettage. She is a
Jehovah's Witness. You consider administering intravenous desmopressin in an attempt to reduce
haemorrhage. Which of the following statements regarding desmopressin is FALSE?

A. it is a synthetic substance and is acceptable to Jehovah's Witnesses
B. it is likely to reduce haemorrhage in this patient
C. it should be given 30 minutes prior to surgery as an infusion
D. its duration of effect is approximately 5 days
E. the intravenous dose is 0.3 mcg.kg-1
A - Clearly true, acceptable to JWs
B - Useful for type 1 vWD, so it is likely to reduce bleeding
C - Infuse in 50mL N saline over 30 min, 30 min prior to surgery sounds about right
D - False - elimination half-time 2.5-4.4 hours
E - Correct dose
During transfusion of platelets a patient develops fever, rigors and vomiting
and becomes hypotensive and tachycardic. The most likely diagnosis is
A. ABO incompatibility
B. anaphylaxis
C. bacterial contamination of the platelets
D. leukocyte mediated transfusion reaction
E. viral contamination of the platelets
C (too systemically unwell for D)
Features of the transurethral resection of the prostate (TURP) syndrome include a11 of the
following EXCEPT

A. agitation
B. angina
C. bradycardia
D. nausea
E. tinnitus
E
The estimated risk of infection following percutaneous exposure (needlestick injury) to HIV is
approximately:

A. 1 in 30
B. 1 in 300
C. 1 in 3,000
D. 1 in 30,000
E. 1 in 300,000
B
The most important factor in reducing peri-operative morbidity in diabetic patients
undergoing peripheral vascular surgery is
A. tight control of blood sugar level in the peri-operative period
B. frequent blood sugar level estimations
C. use of regional rather than general anaesthesia
D. stabilisation of co-existing disease
E. the use of an insulin infusion rather than a subcutaneous sliding scale regimen
D
The observed fall in cardiac output induced by carbon dioxide pneumoperitoneum (with intra-
abdominal pressures below 12 mmHg), during laparoscopic cholecystectomy is primarily a result of

A. a fall in venous return to the heart
B. an increase in systemic vascular resistance
C. increased pressure transmitted to intra-thoracic baroreceptors
D. increased pulmonary vascular resistance
E. reflex bradycardia
The observed fall in cardiac output induced by carbon dioxide pneumoperitoneum (with intra-abdominal pressures below 12 mmHg), during laparoscopic cholecystectomy is primarily a result of

A. a fall in venous return to the heart

B. an increase in systemic vascular resistance True - according to Gerges review.

C. increased pressure transmitted to intra-thoracic baroreceptors

D. increased pulmonary vascular resistance

E. reflex bradycardia


Unfortunately, the ranges given in the CEACP article don't quite fit. The examiners however seem to be making the point that at low abdominal pressures the venous return is actually augmented, but at high abdominal pressures it is markedly reduced. The SVR is increased at all pressures however.
Post-operative delirium in the elderly is associated with all of the following factors EXCEPT

A. pre-existing cognitive deficit
B. pre-existing hearing impairment
C. pre-existing visual impairment
D. post-operative urinary tract infection
E. use of general anaesthesia rather than regional anaesthesia
E
Regarding Epidural Abcess - which is WRONG
A. Diagnosis is DEPENDENT on triad of back pain, fever, and paralysis
B. Occurs at a rate of 1:1000-3000 (OR 1:2000 - 1:5000)
C. Worse outcomes if advanced age
D. Usually gram positive cocci
E. Expectant management may be appropriate
A. Diagnosis is DEPENDENT on triad of back pain, fever, and paralysis - false and the ANSWER to CHOOSE: "The early signs and symptoms may be vague, the 'classic' triad of back pain, fever and variable neurological deficit occurred in only 13% of patients by the time of diagnosis, and contributed to diagnostic delay in 75%." (Grewal et al, Epidural Abscesses in BJA 2006 96(3):292-302)
B. Occurs at a rate of 1:1000-3000 (OR 1:2000 - 1:5000) - true: "Estimating the true incidence of a rare complication from such disparate reports is not easy, but there is some suggestion that it might be of the order of 1 in 1000 in surgical, and 1 in 2000 in obstetric, patients." (Grewal et al, Epidural Abscesses in BJA 2006 96(3):292-302)
C. Worse outcomes if advanced age - true: "With every decade increase in age, the likelihood of poor outcome doubled, presumably due to declining health and, possibly, reduced ‘plasticity’ of the spinal cord." (Grewal et al, Epidural Abscesses in BJA 2006 96(3):292-302)
D. Usually gram positive cocci - true: "In the ‘developed’ world the organisms most frequently encountered are Staphylococcus aureus (57–93% of cases), Streptococci (18%) and a variety of Gram-negative bacilli (13%)." (Grewal et al, Epidural Abscesses in BJA 2006 96(3):292-302)
E. Expectant management may be appropriate - true: "It might be assumed that every patient with an epidural abscess should undergo surgery, but 11% of those identified in a major review did not, and another report identified 38 such individuals in case series and reports published between 1970 and 1990... The neurological deficit was unchanged or improved in all these patients except two, who died from sepsis syndrome, suggesting that the results of medical and surgical treatment are equivalent." (Grewal et al, Epidural Abscesses in BJA 2006 96(3):292-302)
Possible mechanisms of postoperative visual loss include each of the following EXCEPT
A. atherosclerosis causing decreasing blood flow to the optic nerve
B. fluid overload
C. Isovolaemic haemodilution
D. Postually induced raised venous pressure
E. Variations in the number of posterior ciliary arteries
C
The most frequently reported clinical sign in malignant hyperpyrexia is
A. arrhythmia
B. cyanosis
C. sweating
D. tachycardia
E. rigidity
D
A l3-year-old boy with Duchenne's muscular dystrophy

A. is at increased risk of malignant hyperthermia
B. is likely to have significant scoliosis
C. is more likely to be cachectic than obese
D. is unlikely to have cardiac involvement
E. may develop rhabdomyolysis when non-depolarising muscle relaxants are used
B
A 4y.o. boy presents for insertion of grommets. His
maternal great-grandfather is know to have had an
episode of malignant hyperthermia. Which of the following
is the strongest evidence that the boy is NOT susceptible to MH?
A. the boy was exposed to halothane at age 2 with no sequelae
B. the boy has recently been shown to have a normal serum creatinine kinase
C. the boy's grandfather has had a negative muscle contracture test for MH
D. the boy's mother has had negative molecular genetic testing for MH
E. there have been NO other episodes of MH in the family despite exposure to known triggers on multiple occasions
C (only test that can rule out MH)
Each of the following is effective in the treatment of pain from acute herpes zoster EXCEPT

A. acyclovir
B. amitriptyline
C. carbamazepine
D. corticosteroids
E. topical aspirin
A. acyclovir - true: "Antiviral agents started within 72 hours of onset of the herpes zoster rash accelerate the resolution of acute pain (U) (Level I), but do not reduce the incidence of postherpetic neuralgia"
B. amitriptyline - true: "Amitriptyline (used in low doses for 90 days from onset of the herpes zoster rash) reduces the incidence of postherpetic neuralgia"
C. carbamazepine - false: Looks like carbamazepine is good for trigeminal neuralgia but not HZ in other distributions
D. corticosteroids - true: "Prednisolone added to acyclovir during HZ resulted in a modest reduction in pain intensity and improved the rate of skin lesion healing for up to 14 days, with no effect on the overall recovery rate at 3 weeks (Wood et al, 1994 Level II). Prednisolone, either as monotherapy or in combination with acyclovir, increased the likelihood of being ‘pain-free’ at 1 month by a factor of 2.3 (95% CI: 1.4 to 3.5), however there was no difference in the rate of skin healing, compared with placebo"
E. topical aspirin - true: "Topical aspirin, topical lignocaine patch or oxycodone controlled release, provide analgesia in herpes zoster"
A strategy shown to reduce the incidence of severe phantom limb pain
is the use of
A. continuous regional blockade using nerve sheath catheters
B. patient controlled analgesia with opioids post-op
C. perioperative ketamine
D. perioperative NSAIDs
E. spinal anaesthesia for the amputation
A. continuous regional blockade using nerve sheath catheters - false: "Infusions of local anaesthetics via peripheral nerve sheath catheters, usually inserted by the surgeon at the time of amputation, are a safe method of providing excellent analgesia in the immediate postoperative period (Pinzur et al, 1996 Level II; Lambert et al, 2001 Level II). However, they are of no proven benefit in preventing phantom pain or stump pain (Halbert et al, 2002 Level I)."
B. patient controlled analgesia with opioids post-op
C. perioperative ketamine - true: "A small observational study found that while the overall incidence of long-term phantom limb pain was similar in patients given ketamine (bolus dose followed by an infusion, started prior to skin incision and continued for 72 hours postoperatively) compared with no ketamine, the incidence of severe phantom limb pain was reduced in the ketamine group (Dertwinkel et al, 2002 Level III-3). Another study looking at the effects of ketamine reported a numerical but not statistically significant difference in the incidence of phantom limb pain at 6 months after amputation (47% in the ketamine group and 71% in the control group) (Hayes et al, 2004 Level II)."
D. perioperative NSAIDs
E. spinal anaesthesia for the amputation
When compared with intra-muscular or subcutaneous opioid regimens, patient controlled analgesia (PCA} with opioids
A. is equally preferred by patients
B. provides better analgesia
C. results in less opioid-related adverse effects
D. results in lower opioid consumption
E. results in shorter hospital stay
B

"IV opioid PCA provides better analgesia than conventional (IM or SC) opioid regimes."
"no differences in opioid consumption, duration of hospital stay or opioid-related adverse effects."
"Patient preference for IV PCA was significantly higher ... although there was no difference in patient satisfaction"
Which of the following statements regarding the use of epidurally administered adjuvant drugs with epidural analgesia for acute postoperative pain is FALSE?
A. adrenaline added to the local anaesthetic improves thoracic epidural analgesia
B. clonidine added to epidural opioids improves analgesia
C. clonidine prolongs the effects of epidural local anaesthetics
D. ketamine added to opioid based epidural analgesia improves analgesia
E. neostigmine combined with an epidural opioid reduces the dose of opioid required
B or C
Complications of mediastinoscopy include all of the following EXCEPT

A. air embolism
B. cardiac laceration
C. pneumothorax
D. recurrent laryngeal nerve palsy
E. tracheal compression
A. air embolism - TRUE listed in both Miller and CEACCP article
B. cardiac laceration - FALSE would've thought it was a complication but not listed in either Miller or CEACCP. Possibly point of entry too high for it to be a major issue?
C. pneumothorax - TRUE listed in both Miller and CEACCP article
D. recurrent laryngeal nerve palsy - TRUE listed in both Miller and CEACCP article
E. tracheal compression - TRUE listed in Miller
A healthy female patient is undergoing a laparoscopic sterilisation under a relaxant based general anaesthetic.
Which of the following monitors does NOT have
to be in continuous use?
A. Capnograph
B. Electrocardiogram
C. Oximeter
D. Oxygen analyser
E. Ventilator disconnect alarm
ECG: correct "must be available for every anaesthetised patient"
Pulse oximeter "A pulse oximeter must be in use for every anesthesied patient"
Breathing system disconnection or ventilation failure alarm: "must be in continuous operation"
Oxygen analyser: "must be in continuous operation for every patient when an anaesthesia delivery system is used"
Carbon dioxide monitor: "must be in use for every patient under GA"
The single best predictor of difficult intubation in a morbidly obese patient is
A. body weight
B. history of snoring
C. Mallampatti score
D. neck circumference
E. thyro-mental distance
D

"Severe OSA and neck circumference greater than 44 cm are associated with possible difficult intubation
in patients presenting for bariatric surgery. Hence it is important to have an array of the difficult
airway equipment when dealing with these cases in addition to a careful preoperative assessment and
planning with regards to airway management."
Which of the following statements regarding infection control is FALSE ?
A. devices to be used in the upper airway that may cause bleeding must remain sterile until used
B. provided there is an adequate filter between the patient and the breathing circuit, the circuit
can be re-used for subsequent patients on an operating list
C. when performing central neural blockade, the anaesthetist must adopt a full aseptic technique
D. when performing central venous cannulation, the anaesthetist must adopt a full aseptic technique
E. when performing vascular cannulation, the anaesthetist must wash hands and should wear gloves
A
During elective major vascular surgery the best way to reduce the risk of acute renal failure is
to maintain a normal

A. central venous pressure
B. mean arterial blood pressure
C. renal blood flow
D. systemic vascular resistance
E. urine output
B or C
Intubation of patients with acute C5 spinal cord injury

A. can only be safely performed using awake fibreoptic intubation
B. is necessary in most patients for secretion clearance
C. is necessary in most patients to avoid atelectasis
D. is associated with a low risk of aspiration
E. may be safely facilitated by the use of suxamethonium
E
A 25-year-old 80 kg male with no other health problems is undergoing ECT
(electroconvulsive shock therapy) for severe depression. Anaesthesia for
his first 2 treatments consisted of thiopentone 350 mg and suxamethonium
50 mg. The treating psychiatrist is concerned at the limited duration of
seizure activity with treatment despite maximal seizure stimulus. An
acceptable seizure duration would be best be achieved by:
A. adjunctive use of remifentanil to reduce the dose of induction agent
B. clonidine premedication
C. hypoventilating the patient to reduce seizure threshold
D. pretreatment with lignocaine to reduce seizure threshold
E. using propofol instead of thiopentone for induction of anaesthesia
A
Which of the following is NOT an absolute contraindication to magnetic resonance imaging?

A. cardiac pacemaker
B. cerebral aneurysm clips
C. cochlear implant
D. implanted defibrillator
E. prosthetic heart valve
E
You have anaesthetised a patient with a partially obstructing right main bronchus tumour.
You are using Heliox (28% O2:72% He) and sevoflurane. You secure the airway with
an endotracheal tube and continue with the same gas mixture. After 20 minutes the ETCO2 is 28mmHg.
The capnogram waveform is normal. This probably means the patient is
A. Appropriately ventilated with a low cardiac output
B. Appropriately ventilated with a normal cardiac output
C. hypothermic (34C)
D. Hyperventilating
E. Hypoventilating
B correct

Capnography is considered essential in the management of mechanically-ventilated patients. Helium, as an adjunct to mechanical ventilation, is the subject of renewed interest and used increasingly. However, helium affects the performance of infrared capnometry. We constructed a simple device to generate variable mixtures of helium, oxygen and carbon dioxide within the normal physiological range, and tested the performance of two side-stream and one in-line capnographs. We found that addition of helium to the gas mixture caused all three capnographs to underestimate the concentration of carbon dioxide. The underestimation increased as the proportion of helium increased. The maximum underestimation (30%) occurred in a 79:21 helium ⁄ oxygen mixture.

Also, the answer is conveniently

40mmHg x 70% = 28mmHg

which is exactly rhe same as Answer B.
Isoflurane is administered in a hyperbaric chamber at 3 atmospheres absolute pressure using a
variable bypass vaporizer.
At a given dial setting and constant fresh gas flow, vapour will be produced at:
A. the indicated vapour concentration
B. three times the indicated vapour concentration
C. one third the partial pressure obtained at 1 atmosphere
D. the same partial pressure as is obtained at 1 atmosphere
E. three times the partial pressure obtained at 1 atmosphere
The key point is that vaporisers use saturated vapour pressure, which is a function of temperature and not ambient pressure. The concentration delivered is inversely proportional to the ambient pressure (hence one third the indicated vapour concentration - A and B are wrong). "However, the partial pressure pressure of the agent, which determines the clinical effect, remains constant" (Ward's anaesthetic equipment, 5th edition, p491). Hence the answer must be D
In a rotameter the:
A. Bobbin spins inside a tube that has parallel sides
B. Flow is laminar at high flow rates
C. Height of the bobbin is proportional to the pressure drop across the bobbin
D. Pressure drop across the bobbin is constant at varying flows
E. Resistance increases with increasing gas flow
A. Bobbin spins inside a tube that has parallel sides - false - tube is tapered
B. Flow is laminar at high flow rates
C. Height of the bobbin is proportional to the pressure drop across the bobbin
D. Pressure drop across the bobbin is constant at varying flows - true:

"The Rotameter and the Wright peak flow meter are examples of variable orifice flowmeters. Constant pressure in the former is provided by the pressure of the bobbin; the pressure is the ratio of the product of the mass and gravity over the cross sectional area of the bobbin (p = F/A = m a/A). The Rotameter is tapered, and thus the magnitude of the orifice around the bobbin corresponds to the magnitude of gas flow." (Basic science for anaesthetists).

E. Resistance increases with increasing gas flow
An anaesthetised patient is ventilated and has standard monitoring plus a central venous line. As the surgeon is commencing the case, the line isolation
monitor (LIM) alarms indicating a potential leakage current of greater than 5 milliamps from one of the power circuits in use.
The most appropriate action is to
A.check the diathermy return plate
B.disconnect the central line to electrically isolate the patient till the fault is identified
C.make sure the patient is properly "earthed" or "grounded"
D.sequentially unplug non-vital equipment from the circuit until the fault is identified
E.suspend the operation and move the patient to a safe environment
D
The treatment LEAST likely to be useful for torsades de pointes is

A. defibrillation
B. procainamide
C. magnesium
D. electrical pacing
E. isoprenaline
A. defibrillation - true: "Direct current cardioversion usually will transiently terminate the arrhythmia." (Complications during vascular surgery: basic principles and management of arrhythmias, in: Bailliere's Clinical Anaesthesiology Vol. 14, No. 1, pp. 87-96, 2000)
B. procainamide - false and answer to choose: "The anti-arrhythmic drugs that most commonly produce this rhythm are the Vaughan Williams class IA drugs such as quinidine, procainamide and some of the class III drugs, notably ibutilide and sotolol."
C. magnesium - true: "Numerous reports have documented the efficacy of magnesium for this arrhythmia."
D. electrical pacing - true: "Treatment is directed at increasing heart rate; isoproterenol has been used for this purpose, as has overdrive ventricular pacing"
E. isoprenaline - true: See D
In the management of torsades de pointes (polymorphic ventricular tachycardia), all the
following drugs may be useful EXCEPT

A. amiodarone
B. isoprenaline
C. [[lignocaine]
D. magnesium
E. phenytoin
A. amiodarone - false and answer to choose: Amiodarone prolongs the QT interval and is widely reported as a precipitant of torsades de points. "We present two patients who had life-threatening arrhythmias, which are highly likely to be secondary to amiodarone. This class III anti-arrhythmic is commonly prescribed for the acute presentation of supra-ventricular and ventricular arrhythmias. However, occasionally its use can transform arrhythmias from benign to dangerous. These cases highlight the need for careful attention to the indications, cautions and contra-indications of amiodarone as well as the need for vigilance following initiation of anti-arrhythmic therapy." (Resuscitation, Volume 76, Issue 1, Pages 137-141 (January 2008))
B. isoprenaline - true: See above
C. lignocaine - true:

"Class IB antiarrhythmic drugs, such as lidocaine and phenytoin, shorten the action potential duration and, based upon small case series, may be effective in the acute management of TdP and ventricular fibrillation [41,96-99]. They appear to be less predictably effective than pacing or isoproterenol" (Uptodate)

D. magnesium - true: See above
E. phenytoin - true:
Characteristic cardio-pulmonary effects of pulmonary thrombo-embolism include

A. hypoxaemia due to excess perfusion of lung units with a low V/Q ratio
B. hypercarbia due to an increase in physiological dead-space
C. reverse splitting of the second heart sound
D. an increase in compliance of the left ventricle
E. an increase in coronary blood flow to the right ventricle during systole
A
A 24 year old female with mitral valve prolapse develops atrial flutter during a
diagnostic laparoscopy. The drug most likely to revert this arrhythmia is
A. Digoxin
B. Amiodarone
C. Verapamil
D. Esmolol
E. Adenosine
B
Reverse splitting of the 2nd heart sound is caused by
A. acute pulmonary embolism
B. ASD
C. complete LBBB
D. severe MR
E. pulmonary HT
A. acute pulmonary embolism
B. ASD
C. complete LBBB - true: "The most common cause of reversed splitting is complete LBBB, which can be caused either by delayed activation of the LV, as seen in isolated proximal block, or to prolonged mechanical systole (primarily isovolumic contraction time), as seen in proximal or peripheral block invariably associated with significant LV dysfunction" (Hurst's The Heart Ch 12)
D. severe MR
E. pulmonary HT
60 y.o with acute aortic regurgitation. In pulmonary oedema. BP 160/90.
HR was about 90. What is the best treatment pre-op:
A. Intra aortic balloon pump
B. Dobutamine infusion
C. Sodium nitroprusside infusion
D. Beta blocker
E. Dopamine infusion
C
When viewing the central venous pressure trace:
A. an accentuated "a" wave supports the diagnosis of atrial fibrillation
B. a steeper than normal "x" descent supports the diagnosis of tricuspid regurgitation
C. blunted "a" and "v" waves are associated with extensive right ventricular infarction
D. flattened "x" and "y" descents are associated with pericardial constriction
E. a monophasic pattern with obliteration of the "y" descent supports the diagnosis of pericardial tamponade
A - False

In atrial fibrillation, a waves will be absent, and in atrioventricular dissociation, a waves will be dramatically increased ("cannon waves") as the atrium contracts against a closed tricuspid valve.

B - False

In tricuspid regurgitation, the c wave and x descent will be replaced by a large positive wave of regurgitation as the blood flows back into the right atrium during ventricular contraction

C - False

Prom a and v waves with RV infarction due to (a wave) increased atrial pressure from contraction into stiff RV (kind of like increased RA afterload) and v waves increased as often assoc TR hence increased filling RA.

D - False

Pericardial constriction – tall a and v waves, steep x and y descents

E - True
In cardiac tamponade, all pressure will be elevated, and the y descent will be nearly absent
Clinical features supporting the diagnosis of cardiac tamponade include all of the following EXCEPT

A. equal diastolic pressures of all heart chambers
B. ST segment abnormalities
C. increased venous pressure and cardiac output with fluid loading
D. increased patient comfort in the sitting position
E. impalpable apex beat and soft heart sounds
B
The LEAST desirable position for the tip of a central venous catheter which has been
inserted into the left internal jugular vein is

A. mid-way along the left brachiocephalic vein
B. at the junction of the left brachiocephalic vein and the superior vena cava (SVC)
C. in the SVC at the level of the carina
D. at the junction of the SVC and the right atrium
E. in the right atrium
B
Regarding patients aged 65 years or older with recurrent atrial fibrillation (AF)
A. amiodarone and digoxin have similar efficacy in restoring sinus rhythm
B. patients who have been reverted to sinus rhythm should still remain on warfarin therapy
C. patients who remain in atrial fibrillation with heart rates less than 80 beats per minute do NOT require long term warfarin therapy
D. peri-operative therapy with a beta-blocker will commonly lead to restoration of sinus rhythm
E. restoration of sinus rhythm with electrical DC cardioversion improves long-term survival in comparison to controlling heart rate alone
B
Addison's disease lab features: Which is NOT correct
A. Ca 2.50 mmol/L
B. BSL 12.0 mmol/L
C. Na 128 mmol/L
D. K 6.1 mmol/L
E. Urea 15mmol/L
A. Ca 2.50 mmol/L - true: "Serum sodium may be low; potassium, calcium, and urea nitrogen may be elevated" (Quick answers)
B. BSL 12.0 mmol/L - false and answer to choose: "Fasting blood glucose may be low" (Quick answers)
C. Na 128 mmol/L - true: "Addison’s disease is characterized by fatigue, weakness, anorexia, nausea and vomiting, cutaneous and mucosal hyperpigmentation, cardiopenia secondary to chronic hypotension, hypovolemia, hyponatremia, and hyperkalemia." Stoelting
D. K 6.1 mmol/L - true: See C.
E. Urea 15mmol/L - true: See A
All of the following may be associated with ulcerative colitis EXCEPT

A. cirrhosis
B. iritis
C. psoriasis
D. arthritis
E. sclerosing cholangitis
A or C?
Anaemia in chronic renal failure is characteristically

A. due to haemolysis in the renal vascular bed
B. normochromic and microcytic
C. due to defective haemoglobin synthesis
D. responsive to iron and folate therapy
E. associated with increased 2,3-DPG levels in blood cells
E
In HIV infected patients
A. a CD4 count of 200-400 is typical
B. toxoplasmosis is often a presenting feature
C. cardiomyopathy is a recognised complication
D. regional anaesthesia in obstetrics is contraindicated
E. an epidural blood patch should NOT be used to treat a post-dural puncture headache
C
The thromboelastogram pattern typical of that seen in haemophilia is labelled
http://www.AnaesthesiaMCQ.com/images/MCQ_61_apr03_diagram.png

A. A
B. B
C. C
D. D
E. E
B
Heparin induced thrombocytopenia (HITS)
A. is associated with antibodies to complexes of anti-thrombin 3 (ATIII) and heparin
B. is associated with a more rapid drop in platelet count if the patient has been exposed to heparin within the last 3 months
C. is not associated with the use of low molecular weight heparins
D. results in the maintenance of heparin-dependent antibodies indefintely after their development
E. results in thrombotic complications in most patients
B
Respiratory function in quadriplegics is improved by

A. abdominal distension
B. an increase in chest wall spasticity
C. interscalene nerve block
D. the upright position
E. unilateral compliance reduction
B
An INCORRECT statement regarding the autonomic nervous system is that

A. autonomic dysfunction is a predictor for worse long term survival after
myocardial infarction
B. heart rate responses are primarily mediated through the sympathetic nervous system
C. inhalation anaesthetics all impair autonomic reflex responses
D. autonomic dysfunction is a predictor for haemodynamic instability following
anaesthetic induction
E. low heart rate variability is associated with worse cardiac outcomes following
non-cardiac surgery
B

The sympathetic component is primarily involved in adjusting peripheral vascular tone and plays a lesser role in the reflex regulation of HR and cardiac output. The vagal component is primarily involved in regulating HR.
A 33-year-old chronically spinally injured patient becomes hypertensive and sweaty
during general anaesthesia for urinary sphincterotomy. His level of spinal cord injury
is T4 and it is complete. You consider the diagnosis of autonomic hyperreflexia.
Autonomic hyperreflexia
A. could have been prevented by performing subarachnoid anaesthesia
B. is unlikely with a T4 lesion
C. should be treated by administration of a beta-blocker
D. should be treated by administration of an opioid analgesia
E. will resolve once the surgical stimulus ceases
A

"Spinal anaesthesia...can reliably prevent autonomic dysreflexia and spasm"...Anaesthesia for Patients with Chronic Spinal Cord Injury; Current Anaesthesia and Critical Care; 2001 (12); p154-57
In chronic obstructive pulmonary disease (COPD), the variable most closely associated with prognosis is
A. arterial carbon dioxide partial pressure (PaC02)
B. arterial oxygen partial pressure (Pa02)
C. forced expiratory volume in one second (FEV1)
D. forced vital capacity (FVC)
E. response to bronchodilators
C
Patho-physiological features of patients with morbid obesity include

A. a blood volume:body weight ratio which is similar to that of patients with normal body weight
B. an increased blood pressure and systemic vascular resistance compared to that of patients with
normal body weight
C. decreased gastric motility due to increased gastrin secretion
D. cardiac pathology resulting from excess body mass and increased metabolic demand
E. cardiac pathology resulting mainly from fatty infiltration or fatty change of the heart
A. a blood volume:body weight ratio which is similar to that of patients with normal body weight - false: "Total blood volume is increased in the obese but on a volume/weight basis is less than that in non-obese individuals" (Adams and Murphy, Obesity in anaesthesia and intensive care, BJA 2000; 85:91-108)
B. an increased blood pressure and systemic vascular resistance compared to that of patients with normal body weight - false: "hypertension is presumably caused by an increased cardiac output forced into an unaltered peripheral resistance" (Oberg and Poulsen, Obesity: an anaesthetic challenge, Acta Anaesthesiologica Scandinavica 1996, 40: 191-200)
C. decreased gastric motility due to increased gastrin secretion - false: gastrin is a prokinetic hormone
D. cardiac pathology resulting from excess body mass and increased metabolic demand - true by default
E. cardiac pathology resulting mainly from fatty infiltration or fatty change of the heart - false: "The morbidly obese individual is at risk of a specific form of obesity-induced cardiac dysfunction, although the belief is that this is secondary to fatty infiltration of the heart ('cor adiposum') is no longer valid." (Adams and Murphy)
An INCORRECT statement regarding the management of hypocalcaemia is that

A. Correcting a respiratory or metabolic alkalosis increases the level of ionised calcium
B. Calcium should be administered via a central vein as it is irritant to peripheral veins
C. Acidosis will decrease calcium binding to albumin and therefore increase ionised calcium
D. When calcium alone is NOT sufficient for control of hypocalcaemia, Vitamin D metabolites
can be added
E. Calcium chloride has been shown to be superior to calcium gluconate
A. Correcting a respiratory or metabolic alkalosis increases the level of ionised calcium - true: "Initial therapy (of hypocalcaemia in adults consists of correction of any coexisting respiratory or metabolic alkalosis" (Aguilera and Vaughan, "Calcium and the anaesthetist", Anaesthesia 2000, 55:779-790)
B. Calcium should be administered via a central vein as it is irritant to peripheral veins - true: "Calcium is irritating to veins and should be diluted before administration and all solutions are best administered through a central vein." (Aguilera and Vaughan, "Calcium and the anaesthetist", Anaesthesia 2000, 55:779-790)
C. Acidosis will decrease calcium binding to albumin and therefore increase ionised calcium - true: "Acidosis decreases calcium binding to albumin thus increasing ionised calcium, while alkalosis increases binding (calcium shifts onto the albumin molecule), producing a subsequent reduction in the ionised calcium." (Aguilera and Vaughan, "Calcium and the anaesthetist", Anaesthesia 2000, 55:779-790)
D. When calcium alone is NOT sufficient for control of hypocalcaemia, Vitamin D metabolites can be added - true: "When calcium alone is not sufficient for control of hypocalcaemia, vitamin D metabolites can be added." (Aguilera and Vaughan, "Calcium and the anaesthetist", Anaesthesia 2000, 55:779-790)
E. Calcium chloride has been shown to be superior to calcium gluconate - false and answer to choose: "When equivalent calcium doses are administered, both preparations are equally efficacious in restoring the calcium level to normal ( Table 54-12 ). In this regard, calcium gluconate is especially advantageous in peripheral venous administration because extravasated calcium chloride solution can result in severe tissue destruction." (Miller Ch 54)
In patients with chronic renal failure there is

A. increased calcium absorption
B. increased phosphate excretion
C. increased vitamin D3 production
D. increased osteoclastic activity
E. decreased osteoblastic activity
A. increased Ca absorption - false: "Renal osteodystrophy is a complication of chronic renal failure, reflecting the complex interaction of secondary hyperparathyroidism and decreased vitamin D production by the kidneys. As the GFR decreases, there is a parallel decrease in phosphate clearance and an increase in the serum phosphate concentrations that result in reciprocal decreases in serum calcium concentrations. Hypocalcaemia stimulates PTH secretion, which leads to bone resorption and calcium release. As a result of decreased renal production of Vitamin D by the kidneys, intestinal absorption of calcium is impaired, which also leads to hypocalcaemia, stimulation of PTH release, and bone resorption." (Stoelting's Anesthesia and Coexisting Disease, 5th ed, p.332)
B. increased PO4 absorption - false: see above
C. increased vit D3 production - false: see above
D. increased osteoclastic activity - true
E. decreased osteoblastic activity - false
Which of the following statements regarding patients with ankylosing spondylitis is FALSE?

A. amyloid renal infiltration is rarely seen
B. cardiac complications occur in less than 10% of cases
C. normochromic anaemia occurs in over 85% of cases
D. sacroileitis is an early sign of presentation
E. uveitis is the most common extra-articular manifestation
C - only in about 15%
In Marfan’s Syndrome which is NOT related:
A. If develop aortic disease most likely to be aortic stenosis
B. At risk iliac aneurysm
C. development of mitral valve prolapse is more likely than in general population
D. Cardiac myopathy due to medial cystic necrosis/atrophy/ degeneration
E. Intracranial aneurysm
A: (get AR not AS)

Marfan's Sydnrome is associated with aortic root dilatation causing aortic regurgitation. This is due to defective tensile strength in connective tissue of aorta and heart valves. The patients gets dilation, dissection and rupture of the aorta and prolapse of the heart valves especially mitral valve [Stoelting 4th ed p 534
With respect to intra-arterial cannulation and monitoring

A. following cardiopulmonary bypass, a cannula in the brachial artery will tend to give a more
accurate blood pressure reading than in the radial artery
B. radial artery cannulation is not contra-indicated in Raynaud's Disease
C. the Allen's test is a reliable predictor of the adequacy of collateral flow
D. The axillary artery is not suitable for use in cardiac surgery
E. the infective rate of all cannulation sites is approximately 5%
A. following cardiopulmonary bypass, a cannula in the brachial artery will tend to give a more accurate blood pressure reading than in the radial artery - true
"Two minutes after separation from CPB, clinically important (greater than or equal to 10 mmHg) underestimation of systolic aortic pressures occurred in 17 of 33 (52%) radial artery catheters, while occurring in seven of 33 (21%) brachial artery catheters. Radial artery mean pressure underestimated aortic mean pressure by greater than or equal to 5 mmHg in 21 of 33 (61%) patients two minutes after CPB, while an equivalent aortic-to-brachial artery mean arterial pressure difference occurred in nine of 33 (27%) patients." (J Cardiothorac Anesth. 1989 Feb;3(1):20-6)
B. radial artery cannulation is not contra-indicated in Raynaud's Disease - probably false:
Ischemic complications of radial artery cannulation: an association with a calcinosis, Raynaud's phenomenon, esophageal dysmotility, sclerodactyly, and telangiectasia variant of scleroderma. (Anesthesiology. 1993 Mar;78(3):587-9)
C. the Allen's test is a reliable predictor of the adequacy of collateral flow - false: Unreliable
"A review of the literature concerning ischaemic complications following radial artery cannulation suggests that the Allen test has little predictive validity" (Anaesthesia. 1985 Sep;40(9):896-9.)
D. The axillary artery is not suitable for use in cardiac surgery - unsure, but probably false
E. the infective rate of all cannulation sites is approximately 5% - false: infection rate is lower
Femoral artery cannulation for arterial pressure monitoring
A. is associated with more major complications than radial artery cannulation
B. is frequently unreliable in the immediate post-bypass period
C. should be performed with an approach from above the inguinal ligament
D. should be performed with a large bore cannula to avoid erosion of the vessel wall
E. should not be used for over 24 hours because of the risk of infection
A. is associated with more major complications than radial artery cannulation - false:
"The incidence of this complication in the femoral artery is much lower than that in the radial artery." (Critical Care 2002, 6:199-204)[1]
"When compared with radial artery catheterization, the risk of distal ischemia after femoral artery cannulation may be reduced because of the large diameter of the artery, but atherosclerotic plaque embolization is more likely during initial guidewire and catheter placement." (Miller)
"Incidence rates for major complications such as permanent ischaemic damage, sepsis and pseudoaneurysm formation are low and similar for the radial, femoral and axillary arteries. They occur in fewer than 1% of cases." (Critical Care 2002, 6:199-204)[2]
B. is frequently unreliable in the immediate post-bypass period - probably false:
"On conclusion of CPB, mean femoral artery pressures (70.9 ± 6.7 mmHg) are greater than mean radial artery pressures... Although radial artery pressures are more commonly monitored during cardiac surgery, femoral artery perfusion pressures are more reliable during the initial part of CPB, and routine monitoring of femoral artery pressures may prevent vasoconstrictor use on initiation of CPB." (Femoral artery pressures are more reliable than radial artery pressures on initiation of cardiopulmonary bypass in Journal of Cardiothoracic and Vascular Anesthesia Volume 14, Issue 3, June 2000, Pages 274-276)
C. should be performed with an approach from above the inguinal ligament - false: "The operator must be careful to puncture the femoral artery below the inguinal ligament, thereby limiting the risk of arterial injury causing uncontained hemorrhage into the pelvis or peritoneum, a potentially catastrophic complication." (Miller)
D. should be performed with a large bore cannula to avoid erosion of the vessel wall - unsure: How large is a large bore cannula? Certainly, the femoral catheters are larger and longer. Does that avoid erosion??
E. should not be used for over 24 hours because of the risk of infection - very likely false, unless most ICUs ignore this risk
A 40y.o. man with Marfan's syndrome is to undergo thoraco-abdominal aortic
reconstruction for chronic aortic dissection. An intrathecal catheter is
inserted. The purpose of this is to
A. allow drainage of CSF
B. allow intrathecal admin of metabolic substrates
C. allow intrathecal admin of neuroprotective drugs
D. cool the spinal cord
E. facilitate spinal cord function monitoring
A
For a patient positioned for left lateral for posterolateral incision for thoracotomy
A. right brachial plexus is at risk if shoulder flexed > 90 degrees
B. radial nerve is most common injury
C. flex the non-dependant leg, and straighten the dependant leg
D. place axillary roll under chest to improve surgical exposure
E. the neck should be extended to allow access to the airway
Answer: A? "Compression also plays a predominant role in injury with the patient in the lateral decubitus position when the plexus is compressed against the thorax by the humeral head" From R. J. Sawyer, M. N. Richmond, J. D. Hickey, J. A. Jarrratt (2000)Peripheral nerve injuries associated with anaesthesia, Anaesthesia 55 (10), 980–991.

B is false "Frequently damaged nerves include the branches of the brachial plexus and the ulnar and common peroneal nerves. The radial nerve is damaged infrequently" From B. E. Tuncali, B. Tuncali, B. Kuvaki, O. Cinar, A. Doğan, Z. Elar (2005) Radial nerve injury after general anaesthesia in the lateral decubitus position* Anaesthesia 60 (6), 602–604.

C. I think it is the other way round, ie. flex the dependent leg, and straighten the non dependent leg? But i can't be 100% sure now. Maybe it is true (for recovery position yes, but in surgery....)

D. "An axillary roll is placed to minimize the risk of brachial plexus injury" [1]--Life is Beautiful 07:26, 9 Jul 2007 (EDT)

Re B. Wouldn't the long thoracic nerve (by scalpel) be the most commonly injured, or does this not count?Tommytank 07:09, 20 Jul 2007 (EDT)

C: definitely wrong, should be opposite (I've checked)
Carbon dioxide is the most common gas used for insufflation
for laparoscopy because it

A. is cheap and readily available
B. is slow to be absorbed from the peritoneum and thus safer
C. is not as dangerous as some other gases if inadvertently given intravenously
D. provides the best surgical conditions for vision and diathermy
E. will not produce any problems with gas emboli as it dissolves rapidly in blood
C
In relation to obstetric haemorrhage
A. amniotic fluid embolism is unlikely to present as unexplained haemorrhage
B. cell salvage is too dangerous to be recommended because of potential contaminants
C. coagulopathy after severe placental abruption is ONLY likely if fetal death in-utero occurs
D. intravenous magnesium may facilitate replacement of an inverted uterus
E. placenta percreta can be excluded by ultrasound examination
D
"Uterine relaxation may be necessary to replace the uterus; β-sympathomimetic agents, magnesium, and nitroglycerin all have been used to achieve this goal." (Miller 7th ed Ch 69)
Amniotic fluid embolism

A. can be associated with a mortality rate of 80%
B. has an incidence of 1 in 2000 pregnancies
C. is an uncommon cause of peripartum death
D. is associated with a small chance of complications in survivors
E. only presents during labour or caesarean section
A

A. can be associated with a mortality rate of 80% - true: Morgan in 1979 reported an 86% maternal mortality rate based on 272 cases; Yang reported 89% mortality in 38 cases in 2000 (Supplementary appendix 2. Incidence of amniotic fluid embolism and associated maternal and perinatal mortality around the world, in Conde-Agudelo A, Romero R. Amniotic fluid embolism: an evidence-based review. Am J Obstet Gynecol 2009;201:445.e1-13.)
B. has an incidence of 1 in 2000 pregnancies - false: "The estimated incidence of AFE is 1:15,200 and 1:53,800 deliveries in North America and Europe, respectively"
C. is an uncommon cause of peripartum death - false: peripartum death is itself uncommon, but AFE is not an uncommon cause of it.
D. is associated with a small chance of complications in survivors - false: "The prognosis of a woman experiencing AFE has improved with early diagnosis and prompt and aggressive treatment by a multidisciplinary team.145 Although case fatality rates for AFE have fallen, there is still significant morbidity among survivors. In Clark et al’s national registry, 19 among the survivors, persisting neurological impairment was reported in 61% of women and 50% of infants. In the United Kingdom registry,35 of the 31 women who survived, 6% had persisting neurological impairment, whereas of the 33 infants who survived, 18% developed hypoxic ischemic encephalopathy and 6% developed cerebral palsy."
E. only presents during labour or caesarean section - false: "AFE typically
Best evidence in obstetric anaesthesia supports each of the following
assertions EXCEPT

A. colloid prevents hypotension from regional anaesthesia more effectively
than crystalloid
B. fentanyl added to spinal bupivacaine for caesarean section has no
influence on the incidence of intraoperative nausea
C. high doses of ephedrine (>15 mg) are more likely to cause hypertension
than prevent hypotension
D. in labour, combined spinal-epidural analgesia is associated with faster
onset and greater maternal satisfaction than epidural analgesia
E. in nulliparous women, epidural analgesia in labour, compared with
intravenous opioid analgesia, does not increase caesarean section rate
B

A. colloid effectively prevents hypotension from regional anaesthesia more effectively than crystalloid - true:
"In contrast, studies have consistently shown that prehydration using colloid solutions in volumes of 500-1000 mL and 15 mL/kg are more effective than crystalloid (Dahlgren et al 2005, Level II; French et al 2000, Level II; Riley et al 1995, Level II; Siddik et al 2000, Level II) and more effective than no prehydration (Ngan Kee et al 2001a, Level II). However, the greater efficacy of colloids has to be balanced against greater expense and greater risk of adverse effects such as allergies (Cyna et al 2006, Level I)." (ANZCA Obstetric anaesthesia: Scientific evidence)
And think about this: if crystalloid not effective, why do we use them?
B. fentanyl added to spinal bupivacaine for caesarean section has no influence on the incidence of intraoperative nausea - false in a small study of 30 patients where it decreased nausea:
"In summary, our study suggests that 20 µg IT fentanyl is superior to 4 mg IV ondansetron for the prevention of perioperative nausea during cesarean delivery performed with bupivacaine spinal anesthesia." (Anesth Analg. 2000 May;90(5):1162-6.Intrathecal fentanyl is superior to intravenous ondansetron for the prevention of perioperative nausea during cesarean delivery with spinal anesthesia.)
C. high doses of ephedrine (>15 mg) are more likely to cause hypertension than prevent hypotension - probably true:
"Ephedrine has been administered intramuscularly to prevent maternal hypotension during spinal anaesthesia.[23.] Although intramuscular ephedrine effectively maintained arterial blood pressure, prophylactic ephedrine caused significant hypertension in some patients with an associated decrease in fetal pH" (Best Practice & Research Clinical Anaesthesiology Volume 17, Issue 3, September 2003, Pages 377-392 Spinal Anaesthesia)
D. in labour, combined spinal-epidural analgesia is associated with faster onset and greater maternal satisfaction than epidural analgesia - part true, part false:
"When CSE is compared with epidural, both provide effective labour analgesia. CSE has a slightly faster onset of effective analgesia from time of injection but is associated with more pruritus. No studies have meaningfully evaluated the time taken from maternal request to the onset of analgesia. There are no differences in maternal satisfaction, maternal hypotension, mobilisation in labour, mode of birth, incidence of post dural puncture headache or blood patch or neonatal outcome (Simmons et al 2007, Level I)." (ANZCA Obstetric anaesthesia: Scientific evidence)
E. in nulliparous women, epidural analgesia in labour, compared with intravenous opioid analgesia, does not increase caesarean section rate - true:
"Regional analgesia in early labour does not increase the rate of caesarean delivery, provides better analgesia and results in a shorter duration of labour than systemic analgesia (Wong et al 2005, Level II)." (ANZCA Obstetric anaesthesia: Scientific evidence)
Regarding non-obstetric abdominal laparoscopic surgery during the second trimester of pregnancy
A. carbon dioxide pneumoperitoneum induces foetal acidosis
B. fetal heart rate is depressed if maternal intra-abdominal pressure reaches 12 mmHg
C. mechanical ventilation during general anaesthesia should be used to maintain a maternal arterial PaCO2 of 40 mmHg
D. premature labour is a common complication unless prophylactic tocolytics are used
E. the risk of miscarriage or premature labour is NOT increased
A
A 46 year old brewery worker has cirrhosis of the liver with oesophageal varices and
has bled from these varices on one occasion. A portacaval shunt is being considered
as definitive treatment for his portal hypertension. The operation would be contraindicated
if he had

A. a serum bilirubin greater than 50 micromol.litre-1
B. a serum albumin less than 30 gram.litre-1
C. ascites
D. all of the above are true
E. none of the above are true
?E
The LEAST effective means of preventing post-operative deep venous thrombosis
(DVT) in elderly patients undergoing surgery for a fractured neck of femur is the use of

A. intermittent pneumatic compression devices
B. low molecular weight heparin (LMWH) with no haematologic monitoring
C. spinal anaesthesia
D. unfractionated heparin adjusted to upper range normal aPTT
E. warfarin adjusted to an INR of 1.5 times normal
??
The most effective method of deep venous thrombosis (DVT) prophylaxis for a fifty-year-old
woman presenting for anterior resection for cancer of the colon would be

A. electrical calf stimulation
B. Dextran 70 infusion
C. graduated compression stockings
D. intermittent pneumatic leg compression
E. low dose heparin (5000 units bd)
E
In regards to laparotomy and hemicolectomy, the least effective way to minimize post op surgical infection is:
A. Aggressive peri-operative fluid management
B. Timely administration of prophylactic antibiotics
C. Perioperative hyperoxia
D. Avoidance of hypothermia
E. Avoidance of blood transfusion
A
Pneumoperitoneum for laparoscopy is commonly associated with an INCREASE in each of the following EXCEPT

A. arterial pressure
B. inotropic state
C. secretion of vasopressin
D. systemic vascular resistance
E. venous resistance
A. arterial pressure - does increase commonly
B. inotropic state - ?False - probably best answer. (See Miller 7th Edn, p2190 - the figure says decreased inotropism (though with 2 question marks suggesting significant uncertainty!)
C. secretion of vasopressin - definitely increases (Miller again)
D. systemic vascular resistance - increases
E. venous resistance - increases
45 y.o for elective laproscoptic choecystectomy. No recent hx cholelithiasis.
Drinks 40g of alcohol per week and smokes 40/day. URTI 1/52 ago. Finishing
course of augmentin. No respiratory symptoms now. LFTS done – ALP 300, GGT 300,
ALT normal, AST normal, albumin normal. What is next step?
A. LFTS changes likely due to recent illness. Proceed with case
B. Defer case until liver USS done
C. Cancel case as patient has early cirrhosis
D. Perform hepatitis screen pre-operation
E. likely secondary to drugs, so just continue
B: True

To exclude CBD pathology that will change his management
Patient presents with carcinoid syndrome and developes hypotension intraoperatively. Best drug to treat it is:
A. Noradrenaline
B. Adrenaline
C. Metaraminol
D. Octreotide
E. Ephedrine
D
Each of the following statements regarding vasospasm occurring after SAH is true EXCEPT:
A. cerebral vessel narrowing, demonstrated angiographically, can be reversed 30-40% of the time by administering nimodipine
B. combination therapy resulting in HT, hypervolaemia and haemodilution is a mainstay for prevention and treatment
C. other cause of neurological deterioration (such as hydrocephalus) need to be excluded before making the diagnosis
D. sequential TC doppler measurement may detect those patients at risk
E. the peak incidence is 7-10 days after the SAH
A
Post grade II SAH in 50 year old woman who has just returned from successful endoluminal
coiling. The best thing to include in subsequent management to prevent vasospasm would be:
A. at least 3 L of IV normal saline per day
B. IV or oral magnesium supplementation
C. maintain systolic blood pressure above 160mmHg
D. Keep intubated and ventilate for 24 hours before de-sedating
E. cool to 33 degrees
???
Each of the following statements concerning surgery for correction of scoliosis is true EXCEPT

A. one third of the blood loss occurs postoperatively
B. major blood loss is frequently accompanied by a consumptive coagulopathy
C. surgery will halt progression of the restrictive lung deficit
D. the major neurological deficits that occur are usually due to damage to the posterior
columns of the spinal cord
E. the use of aprotinin reduces blood loss
D
The absorption of fluid into the circulation during transurethral prostatectomy (TURP)
is NOT related to

A. prostate size
B. height of the irrigation fluid bag
C. duration of surgery
D. surgical technique
E. type of irrigation fluid
E
The absorption of fluid into the circulation during transurethral prostatectomy (TURP)
is NOT related to

A. prostate size
B. height of the irrigation fluid bag
C. duration of surgery
D. surgical technique
E. type of irrigation fluid
A. prostate size - true: "Large prostatic glands have rich venous networks that promote intravascular absorption of irrigation solution" (Yao and Artusio's Anesthesiology, 6ED, p.808)
B. height of the irrigation fluid bag - true: "The hydrostatic pressure of the irrigation solution is an important determinant of the solution absorption rate" (Yao and Artusio's Anesthesiology, 6ED, p.808)
C. duration of surgery - true: If the solution is absorbed at a constant rate, the longer the operation, the greater the amount absorbed
D. surgical technique - true: "The violation of the prostatic capsule during surgery promotes entry of irrigation solution into the periprostatic and retroperitoneal spaces." (Yao and Artusio's Anesthesiology, 6ED, p.808)
E. type of irrigation fluid - false and the answer to choose: As described below, fluid is absorbed through the venous networks rather than across a semi-permeable membrane. The makeup of the fluid DOES affect the likelihood of TURP syndrome as it may alter electrolyte homeostasis, BUT it will not affect the volume of fluid absorbed.
Lowering intra-ocular pressure by applying pressure to the globe (e.g. Honan balloon)
is typically contraindicated in a patient having

A. a revision corneal graft
B. a revision trabeculectomy
C. an extra-capsular lens extraction
D. a redo vitrectomy
E. repeat retinal cryotherapy
B
In the recovery room, following general anaesthesia for renal transplant surgery, your patient is
found to have a serum potassium concentration of 6 mmol.l-1, despite having a normal potassium
concentration pre-operatively. His oxygen saturation is 96% on approximately 40% oxygen via a
Hudson mask. He is still unconscious, but breathing spontaneously at 8 breaths per minute. The most
likely cause of his hyperkalaemia is

A. beta-blockers which he received peri-operatively
B. catabolic stress of surgery
C. opioid induced narcosis causing carbon dioxide retention
D. renal graft failure
E. washed red blood cell transfusion, which he received intraoperatively
A. beta-blockers which he received peri-operatively - false
B. catabolic stress of surgery - unlikely
C. opioid induced narcosis causing carbon dioxide retention - most likely
D. renal graft failure - unlikely as the graft probably hasn't even taken yet
E. washed red blood cell transfusion, which he received intraoperatively - maybe, but washed red cells have a lower potassium content
Following a cadaveric renal transplant under general anaesthesia, your patient's plasma K+ increases
to 6.0 mmol.l-1 in recovery after being 5.0 mmol.l-1 pre and intra-operatively. This patient requires

A. an intravenous infusion of CaCl2 (10 mls over 20 minutes)
B. arterial blood gases to ascertain the acid/base status
C. potassium exchange resins rectally
D. sodium bicarbonate infusion (50- 100 mEq over 5- 10 minutes)
E. urgent haemodialysis
B

A. an intravenous infusion of CaCl2 (10 mls over 20 minutes)
B. arterial blood gases to ascertain the acid/base status - probably the answer, after wading through all the comments below
C. potassium exchange resins rectally
D. sodium bicarbonate infusion (50- 100 mEq over 5- 10 minutes)
E. urgent haemodialysis
In congenital diaphragmatic hernia

A. there is hyperplasia of pulmonary arterioles in the hypoplastic lung
B. right-sided lesions are more pathologically significant
C. vasodilator drugs are contraindicated
D. right-sided lesions through the foramen of Bochdalek are the most common
E. intrapulmonary shunts are the major cause of cyanosis
A. there is hyperplasia of pulmonary arterioles in the hypoplastic lung - true: "Intrapulmonary arteries become excessively muscularized during gestation with thickened adventitia and media; moreover, muscularization extends more peripherally" (King and Booker, Congenital diaphragmatic hernia in the neonate, Continuing Education in Anaesthesia, Critical Care & Pain, Volume 5 Number 5 2005)
A 4-year-old child with obstructive sleep disorder presenting for tonsillectomy

A. is likely to suffer from daytime somnolence
B. is unlikely to have a history of snoring
C. is suitable for day-case surgery
D. has a 40% chance of postoperative vomiting without antiemetic treatment
E. is likely to lose 5% of their blood volume during surgery
D - too low (up to 70%)
E - ?too high (64mL blood loss)
Spinal anesthesia in infants

A. eliminates the risk of postoperative apnea
B. has a lower failure rate than in adults
C. lasts for a shorter time than in adults
D. may be performed at a higher level than in adults
E. often causes hypotension if the infant is awake
C
CEACCP "local and regional anaesthesia in infants"

Proven to reduce risk of apnoeas
Failure rate of 20%
Duration 20-60 minutes (the answer)
Spinal cord terminates more inferiorly (L3)
The peak effect of oral midazolam as a premedication in children occurs after

A. 10 - 15 minutes
B. 20 - 30 minutes
C. 35 - 45 minutes
D. 50 - 60 minutes
E. 65 - 75 minutes
20 - 30 minutes - true: "The dose of oral midazolam... usually results in a satisfactorily sedated child in approximately 10-15 min with a peak effect occurring at approximately 20-30 min, with minimal to no delay in recovery, even for brief procedures." (Cote, Preoperative preparation and premedication, BJA 1999, 83:16-28)
Which of the following statements regarding pre-operative oral midazolam in children (in
appropriate dosage) is INCORRECT?

A. Distress at induction is reduced by approximately one half
B. Peak effect is at 20 - 30 minutes
C. Pre-operative anxiety is reduced
D. Post-operative maladaptive behaviour is reduced
E. Significantly delays recovery and discharge
?D
A 20 kilogram child has a haemoglobin of 60 g.l-1. The child is normovolaemic and there is no
on-going blood loss. The volume of packed cells required to increase the haemoglobin level to
100 g.l-1 is

A. 80 ml
B. 160 ml
C. 320 ml
D. 500 ml
E. 750 ml
A simple solution is to use Frank Shann's equation:

4ml/kg of packed cells will increase Hb by 10g/L.

Thus 4x20x4 = 320 mls (which is option C).
Pneumonia in children

A. if staphylococcal, is commonly associated with a pleural effusion
B. is commonly caused by mycoplasma pneumoniae in young children
C. is most commonly caused by streptococcus pneumoniae
D. is rarely caused by chlamydia trachomatis in neonates
E. often presents as lethargy without fever or cough
A
Regarding endotracheal tubes for paediatric patients

A. a 2.5 mm endotracheal tube is the appropriate size for a term neonate
B. armoured (wire spiral) endotracheal tubes have the same outside diameter as non-armoured
endotracheal tubes (of the same internal diameter)
C. the outside diameter (in mm) of an appropriately sized tube is given by the formula (Age/4) +4
D. the same diameter tube is used for nasal and oral intubation in a child
E. uncuffed, paediatric endotracheal tubes do NOT have a Murphy’s eye
A. False - term neonate would be 3.0-3.5 - preterms would be 2.5 tubes

B. False - armoured tubes are thinner, so for the same internal diameter, armoured tubes are smaller

C. False - this formula refers to the INTERNAL diameter.

D. TRUE - albeit at different lengths of insertion

E. False - they do have a Murphy eye!
The glomerular filtration rate of a paediatric patient (in ml.min-1.m-2) is
similar to that of an adult by the age of approximately

A. 1 month
B. 6 months
C. 2 years
D. 4 years
E. 6 years
C
A one year old child arrests with VT. Has had 2x DC shocks, and 100 mcg adrenaline.
Further 1x DC shock given. What is next step?

A. 20 J DC shock
B. 40 J DC shock
C. 50 mg amiodarone
D. 100 mcg adrenaline
E. 1000 mcg adrenaline
C

In VF/VT:

Defibrillate the heart: Give 1 shock of 4 J kg-1 if using a manual defibrillator.
Resume CPR: Without reassessing the rhythm or feeling for a pulse, resume CPR immediately, starting with chest compression.
Continue CPR for 2 min.
Pause briefly to check the monitor: If still VF/VT, give a second shock at 4 J kg-1 if using a manual defibrillator
Resume CPR immediately after the second shock.
Consider and correct reversible causes (see above: 4Hs and 4Ts).
Continue CPR for 2 min.
Pause briefly to check the monitor: If still VF/VT: Give adrenaline 10 microgram kg-1 followed immediately by a (3rd) shock.
Resume CPR immediately and continue for 2 min.
Pause briefly to check the monitor: If still VF/VT: Give an intravenous bolus of amiodarone 5 mg kg-1 and an immediate further (4th) shock.
Continue giving shocks every 2 min, minimising the breaks in chest compression as much as possible.
Give adrenaline immediately before every other shock (i.e. every 3-5 min) until return of spontaneous circulation (ROSC).
Newborn with diaphragmatic hernia. Initially sats 95% on RA. Now beginning
to develop respiratory distress. Next appropriate step:
A. Awake intubation
B. Trial NO prior to intubation and ventilation
C. Rapid sequence induction
D. Mask ventilation throughout case with CPAP
E. Gas down with CPAP
A
You are commencing general anaesthesia for a 2-year-old child to allow biopsy of an anterior mediastinal mass, A pre-operative CT scan demonstrated
compression of the lower trachea and the carina by Ihe mass. During inhalational induction, the child desaturates to 70% due to airway compression
by the mass. You should
A. apply continuous positive airway pressure (CPAP) via facemask
B. arrange urgent median sternotomy
C. intubate the patient and allow spontaneous ventilation
D. intubate the patient and provide positive pressure ventilation
E. place the patient in the prone position
E
With respect to gastric volumes and fasting in children,
A. casein-predominant milks empty faster than whey-predominant milks
B. children have a higher incidence of aspiration than adults
C. solids rely on first order kinetics for gastric emptying but liquids follow zero order kinetics
D. the rate of gastric emptying is NOT related to the energy content of the meal
E. unlimited clear fluid ingestion 2 hours before surgery does NOT affect volume, but does affect the pH of stomach contents
B
The nerve which supplies the lobule of the ear is the

A. greater auricular
B. lesser occipital
C. auricular branch of the vagus
D. auriculo-temporal
E. greater occipital
great auricular - true: "The great auricular nerve (C2,3) is the largest cutaneous branch of the cervical plexus. It hooks around the mid-point of the posterior border of sternocleidomastoid, then passes across it in the direction of the angle of the mandible. On this muscle it breaks up into three terminal branches.

1 Auricular - supplying the lower two-thirds of the medial aspect of the external ear and the lateral surface of the lobule.
2. Mastoid - to the skin over the mastoid process.
3 Facial - to the skin over the masseter and the parotid gland." (Anatomy for anaesthetists p.148)
The innervation of the human larynx is such that

A. the internal laryngeal branch of the superior laryngeal branch of the vagus supplies
the lingual surface of the epiglottis
B. in the cadaveric position the cords are fully abducted
C. the recurrent laryngeal nerve supplies all the intrinsic muscles of the larynx
D. the glossopharyngeal nerves are sensory to the laryngeal mucous membrane above the level
of the vocal cords
E. cord paralysis can be produced by a distended endotracheal cuff in the larynx compressing
a branch of the recurrent laryngeal nerve against the thyroid cartilage
E. cord paralysis can be produced by a distended endotracheal cuff in the larynx compressing a branch of the recurrent laryngeal nerve against the thyroid cartilage - true: "Compression of the anterior branch of the recurrent laryngeal nerve between the cuff of the endotracheal tube and the posterior part of the thyroid cartilage was the likely mechanism" (of paralysis) [Anaesth Intensive Care. 2004 Jun;32(3):417-8. Bilateral adductor vocal cord paresis following endotracheal intubation for general anaesthesia.]
The muscles of the larynx which separate the vocal cords are the

A. inter-arytenoids
B. lateral crico-arytenoids
C. oblique arytenoids
D. posterior crico-arytenoids
E. thyro-arytenoids
D. posterior cricoarytenoid - true: "The posterior cricoarytenoid muscle arises from the posterior surface of the lamina of the cricoid and is inserted into the posterior aspect of the muscular process of the arytenoid. It abducts the cord by external rotation of the arytenoid and thus opens the glottis; it is the only muscle to do so." (Ellis, Anatomy for Anaesthetists, 8ED, p34)
Stellate ganglion is where:
A. at the level of the body of C6 (spine of C6)
B. posterior to the brachial plexus sheath
C. anterior to the dome of the pleura
D. anterior to the thoracic duct
E. anterior to scalenius anterior
E.
Relations of stellate ganglion Anaesthesia UK website

Anterior
The structures anterior to the ganglion include the skin and subcutaneous tissue, the sternocleidomastoid and the carotid sheath. The dome of the lung lies anterior and inferior to the ganglion.
Medial
The prevertebral fascia, vertebral body of C7, oesophagus and thoracic duct lie medially.
Posterior
Structures posterior to the ganglion include the longus colli muscle, anterior scalene muscle, vertebral artery, brachial plexus sheath and neck of the first rib
The following is a view obtained at direct laryngoscopy.
http://www.anaesthesiamcq.com/images/viewatlaryngoscopyQ5_jul07.jpg
According to the classification of Cormack-Lehane, this would be classified as grade:
A. 1
B. 2
C. 3
D. 4
E. 5
Grade 2. p41 Ellis and Feldman. Have a small amount of cords showing.

Pictures of Cormack-Lehane grades and Mallampatti grades

http://www.pdh-odp.co.uk/images/Grades.jpg
Regarding spinal cord blood supply, the
A. anterior spinal artery arises from the posterior inferior cerebellar arteries
B. anterior spinal artery supplies 50% of the spinal cord, while the posterior spinal arteries supply 25% each
C. largest radicular artery is usually in the lumbar region
D. posterior spinal arteries are only supplied by radicular arteries
E. upper cervical segment of the spinal cord receives most of its blood flow from the vertebral arteries
A: False…anterior spinal artery arises from a branch from each vertebral artery

B: False…anterior artery supplies anterior 2/3 of the spinal cord while the posterior arteries supply the remaining 1/3

C: Probably false…most references state it is usually from low thoracic or high lumbar. Wikipedia states in 75% of people it arises from between T9 and T11

D: False…posterior spinal arteries arise from PICA

E: True…anterior spinal artery is from vertebrals, posterior spinal artery is from PICA which comes off vertebrals, and the segmental medullary arteries in the neck branch off the vertebrals as well
The best predictor of poor outcome for a peri-operative ulnar nerve injury is

A. a delay in symptom onset to more than 48 hours postoperatively
B. association with anaesthesia lasting more than 2 hours
C. association with a brachial plexus block
D. presence of bilateral injury
E. presence of mixed sensory and motor deficit
E. presence of mixed sensory and motor deficit - true: "Patients with only sensory deficits 3 months after their procedures had a greater chance of complete recovery at 1 yr than did patients with mixed sensory and motor deficits." (Warner et al, "Ulnar Neuropathy" in Anesthesiology, 1994, 81:1332-1340)
Recognised factors that predispose upper limb nerves to compression, under anaesthesia include

A. forearm extension and supination
B. extreme flexion of elbows across chest
C. internal rotation of abducted arm
D. lateral position - uppermost arm flexed in arm support
E. prone position - arms by side and fingers flexed
B. extreme flexion of elbows across chest - correct: "Injuries may also occur when the nerve is stretched around the medial epicondyle during extreme flexion of the elbow across the chest" (Anaesthesia ume 55, Issue 10, 2000. Pages: 980–991)

"The ulnar nerve, like most peripheral nerves, is intolerant of stretch beyond 10% of its normal length. Fig. 3 illustrates the cubital tunnel retinaculum, which is lax while the forearm is extended but becomes taut as the elbow is flexed [10]. Thus, persistent elbow flexion creates two mechanisms of potential nerve injury: direct internal compression and internal fixation within the cubital tunnel, which renders the remainder of the nerve more vulnerable to stretch along its course." (Anesthesiology Clin N Am 20 (2002) 589–603)
What do C6/7 motor function do
A. flex/extension of fingers
B. flex /extend wrist
C. shoulder ext rotation / abduction
D. elbow pronation/supination
E. flexion at elbow

A c7,8
B true Last p 16
C c5
D Pronation = c7,8 Supination = c5
E c5,c6

You are asked to see a 60 y.o. male 2 days following a
cervical laminectomy because he has new new neurological
symptoms in his right arm. The surgical team think these
may be due to poor patient positioning. The sign that would
most help differentiate c C8-T1 nerve root injury from an
ulnar nerve injury is
A. loss of sensation in the index finger
B. loss of sensation in the little finger
C. weakness of the abductor digiti minimi muscle
D. weakness of the abductor pollicis brevis m
E. weakness of the first dorsal interosseous m.
D. weakness of the abductor pollicis brevis m - true: T1 and ulnar nerve have very similar sensory and motor distribution. HOWEVER, the one difference is that the LOAF muscles are supplied via the median nerve and provide a point of differentiation. See below quotes from Anatomy for Anaesthetists

"The lower cord of the normally constituted brachial plexus may be snared over the fibrous prolongation of an incomplete cervical rib with resultant paraesthesia over the distribution of C8 and T1 (the ulnar border of the forearm and hand), together with weakness and wasting of the small muscles of the hand, especially those of the thenar eminence." (Anatomy for Anaesthetists 8E)

Ulnar nerve "Muscular branches - to flexor carpi ulnaris, the medial half of flexor digitorumprofundus, and all the intrinsic muscles of the hand apart from the lateral two lumbricals and the three muscles of the thenar eminence." (Anatomy for Anaesthetists 8E) Recall: LOAF muscles supplied by the recurrent branch of the median nerve; Lateral two lumbricals, opponens pollicis, abductor pollicis brevis, flexor pollicis brevis
The syndrome known as transient radicular irritation, or transient neurologic symptoms,
following spinal anaesthesia occurs ONLY in patients

A. given intrathecal lignocaine
B. having surgery performed in the lithotomy position
C. given hyperbaric intrathecal solutions
D. who experience an initial full recovery from spinal blockade
E. who experience complete motor blockade with their spinal block
D. who experience an initial full recovery from spinal blockade - true: "Transient neurologic symptoms (TNS) are symptoms of pain in the gluteal region that can radiate down both legs and appear within a few hours to 24 hours after an uneventful spinal anesthetic." (Miller Ch 70)
An 85-year-old woman requires an emergency repair of an
obstructed inguinal hernia. She has recently suffered a
pulmonary embolus and is taking warfarin. You elect to perform an inguinal field block.
Which of the following nerves does NOT need to be blocked?
A. iliohypogastric
B. ilioinguinal
C. ilioinguinal of the contralateral side
D. femoral
E. subcostal
D

the following nerves need to be blocked:

Ilioinguinal nerve
Iliohypogastric nerve
Genitofemoral nerve
Subcostal & thoracic nerves
Need to block innervation from the contralateral side
Block Height required for orchidectomy:
A. xiphisternum
B. nipple
C. umbilicus
D. midway between pubis and umbilicus
E. pubis
Need block to T9/10 (OHA pg 641) = umbilicus (C)

Xiphisternum = T6
Nipple = T4
Midway between pubis & umbilicus = T11/12
Pubis = L1
To achieve maximum anaesthesia with minimal risk of trauma to veins, the tip of a needle used for a
medial peribulbar injection should be advanced no further past the equator of the globe than
A. 5 mm
B. 10 mm
C. 15 mm
D. 20 mm
E. 25 mm
?B
The LEAST likely complication of a peribulbar regional block for cataract surgery is
A. bradycardia
B. damage to the optic nerve
C. globe perforation
D. ocular muscle palsy
E. peribulbar haemorrhage
B?
Following a retrobulbar block of the eye which of the following features would suggest brainstem spread
of the local anaesthetic?
A. an atonic pupil
B. blindness in the blocked eye
C. blindness in the contralateral eye
D. difficulty in swallowing
E. diplopia
D?
The signs of exposure to a nerve agent such as Sarin or VX include

A. bronchodilation
B. dry skin
C. muscle fasciculation
D. pupillary dilatation
E. tetany
D

The main effect of nerve agents is inhibition of acetylcholinesterase (AChE) and butaryl cholinesterase in the cholinergic nervous system.
oisoning is caused by the accumulation of acetylcholine and not by the OP itself. As a result of stimulation of muscarinic synapses, there is miosis; ciliary body spasm causing pain[32]; glandular hypersecretion, including salivary, bronchial, and lacrimal; sweating; cardiac effects, including bradycardia, atrioventricular block, and Q–T prolongation; bronchoconstriction; vomiting; severe diarrhea; and fecal incontinence. The nicotinic effects are manifested by fasciculation and paralysis at the neuromuscular junction.
A 32-year-old patient is admitted with early acute liver failure (unrelated to paracetamol
ingestion). Management should include:

A. avoidance of intubation to monitor encephalopathic progress
B. consideration for liver transplant if the INR (international normalised ratio) is over 3
C. limited use of sodium containing fluids during acute resuscitation
D. N-acetyl-cysteine as a generall hepatoprotective agent
E. prophylactic antibiotics
E. prophylactic antibiotics - true: "Prophylactic antimicrobials with broad-spectrum coverage of gram-positive and gram-negative activity including an anti-fungal (e.g. piperacillin with tazobactam and fluconazole) should be administered on admission, as this halves the incidence of infective episodes when compared with commencement at the time of suspected infection. However, this benefit must be balanced against the risk of developing multi-resistant pathogens." (Continuing Education in Anaesthesia, Critical Care & Pain | Volume 4 Number 2 2004, p40f)
A 25-year-old man, involved in a motor vehicle crash, presented with a GCS (Glasgow Coma Score) of 5. He was intubated and ventilated, and CT scan of his head
was consistent with diffuse axonal injury. An ICP (intra-cranial pressure) monitor has been placed. The patient's ICP has ranged between 15 and 25 mmHg over the
last 2 hours, despite intravenous mannitol and moderate hypocapnia. He has stable haemodynamics with a mean arterial pressure of 95-100 mmHg. He now requires
general anaesthesia to manage his orthopaedic injuries. The most appropriate agents for maintenance of anaesthesia would be
A. isoflurane / remifentanil
B. propofol and fentanyl
C. propofol and nitrous oxide
D. sevoflurane and nitrous oxide
E. sevoflurane and remifentanil
?B
A 30-year-old man presents to the emergency department following a high speed motor vehicle accident. His blood pressure is 70/50 mmHg with a strongly positive
FAST (focused abdominal sonography for trauma). His chest X-¬ray shows a widened mediastinum. The most appropriate method to assess the widened mediastinum in
this patient is
A. aortogram
B. CT angiogram of the chest
C. repeat chest X-ray
D. intraoperative TOE (transoesophageal echocardiography)
E. transthoracic echocardiography
D
A 50-year-old female patient presents with a 12 hour history of feeling unwell and is found to fulfill the criteria
for Systemic Inflammatory Response Syndrome.
Her blood pressure is 80/45 mmHg, her pulse rate is 90 beats.min1 and her central venous pressure is 12 mmHg.
The hypotension is most appropriately managed with
A. adrenaline
B. dobutamine
C. dopamine
D. ephedrine
E. noradrenaline
E
Which one of the following is most likely to be associated with a high mixed venous oxygen saturation (SvO2)?

A. acute myocardial infarction
B. acute pulmonary embolism
C. cardiac tamponade
D. sepsis
E. severe liver disease
D
The best indicator of adequate fluid resuscitation in the trauma patient is:

A arterial pH
B blood pressure
C core temperature
D pulse rate
E serum lactate level
E
When optimising patients for surgery using goal-directed therapy, which of the following parameters is LEAST useful?

A. blood pressure
B. cardiac index
C. oxygen saturation of blood aspirated from a central venous catheter
D. oxygen saturation of blood aspirated from the distal port of a pulmonary artery catheter
E. stroke volume variability
?A
The perioperative use of beta adrenergic antagonists
A. exerts a cardioprotective effect entirely by reduction of heart rates
B. Is best started intra-operatively
C. Is contraindicated in patients with chronic airways limitation
D. Is contraindicated in patients receiving ACEI
E. Is safe in patients with moderately impaired ventricular function
A. exerts a cardioprotective effect entirely by reduction of heart rates - probably false
B. Is best started intra-operatively - false
C. Is contraindicated in patients with chronic airways limitation - maybe false: definitely false if there is no reversible component to the COAD, but if there is, beta-2 receptors cause bronchodilation and their blockage is relatively contraindicated
D. Is contraindicated in patients receiving ACEI - false
E. Is safe in patients with moderately impaired ventricular function - maybe true: bisoprolol and carvedilol are both beta blockers and are used to treat heart failure; however also note that beta-1 receptors increase cardiac contractility.
Beta-blockers and angiotension converting enzyme inhibitors should be used in all patients with a recent or remote history of MI regardless of EF or presence of HF (AHA guidelines)
The correct ranking of fat/blood partition co-efficients, in order of increasing solubility
in fat, for sevoflurane (S), isoflurane (I), desflurane (D) and nitrous oxide (N2O) is
A. N2O < D < S approximately = I
B. N2O approximately = D < S < I
C. N2O < S < D < I
D. D < N2O < S < I
E. D < N2O approximately = S < I
A

Table 1-7 p24 of Stoelting: Nitrous 2.3 Desflurane 27.2 Isoflurane 44.9 Sevoflurane 47.5
The MAC (Minimum Alveolar Concentration) of volatile anaesthetics is DECREASED by

A. hypercarbia (pCO2 55 mmHg)
B. hypernatraemia
C. hyperthermia
D. lithium
E. tricyclic antidepressants
D
Effects of volatile anaesthetic agents on the brain include
A. maintenance of cerebral blood flow when used with hypocapnia
B. uncoupling of autoregulation, with a consequent rise in intracranial pressure
C. reduction of cardiac output and cerebral blood volume when used at concentrations of 1.3 MAC
D. maintenance of cerebral metabolic rate, but reduction of cerebral electrical activity
E. equal depression of all neurons of the brain at 1.3 MAC concentration
Option A - Incorrect. Cerebral blood flow is increased

Option B - Correct. Autoregulation is uncoupled with volatiles, leading to a rise in ICP despite lowering cerebral metabolic rate.

Option C - Incorrect. Cerebral blood volume icnreases.

Option D - Incorrect. Cerebral metabolic rate decreases.

Option E - Incorrect. Not all volatiles depress equally e.g. enflurane is capable of producing excitation and seizures.
Norpethidine toxicity
A. Is only seen if renal function is abnormal
B. Develops because the half life of norpethidine is twice that of pethidine
C. Is not seen unless treatment is prolonged
D. only seen if dose of pethidine in excess of 1.2g/day
E. May manifest early as anxiety and mood changes
E

Norpethidine toxicity

A. Is only seen if renal function is abnormal - false
B. Develops because the half life of norpethidine is twice that of pethidine - maybe: "Although the elimination half-life of pethidine varies from 3 to 6 h, that of norpethidine is around 17 h in healthy patients, but it may be much longer if renal function is compromised."
C. Is not seen unless treatment is prolonged - false: "Numerous cases of seizures have been reported in patients receiving pethidine over a period of days or in some cases only hours."
D. only seen if dose of pethidine in excess of 1.2g/day - false
E. May manifest early as anxiety and mood changes - true: "toxic side effects such as seizures, agitation, irritability, tremors, twitches and myoclonus"
In patients with renal failure each of the following drugs has prolonged
clearance or has active metabolites with prolonged clearance except:
A. Aspirin
B. Buprenorphine
C. Codeine
D. Pethidine
E. Tramadol
B. Buprenorphine - true:

"Mean terminal half-lives are 24 hours following sublingual administration and 2 to 3 hours after parenteral injection; two-thirds of the drug is excreted unchanged, mainly in faeces, while the remaining one third is metabolised predominantly in the liver and gut wall via glucuronidation to an inactive metabolite, buprenorphine-3-glucuronide, and via CYP3A4 to norbuprenorphine, which has 40 times less analgesic effect than buprenorphine (Kress, 2009).
"No dose adjustment is necessary in patients with renal impairment" (Mims online)
Naltrexone
A. given as a single usual dose antagonises the effects of opioids for approximately 8 hours
B. mainly renally metabolised
C. no hepatic side effects even at high doses
D. is mixed opioid agonist-antagonist
E. used for alcohol abuse
E

Duration of action > 24hours (A = false)

"effective half-time for return to baseline opiate receptor occupancy after 50 mg of oral naltrexone to be 72 to 108 hr." [1]

Hepatic metabolism. (B = false)
Naltrexone has the capacity to cause hepatocellular injury when given in excessive doses. Naltrexone PI (C = false)
Opioid antagonist. (D = false)
Used for alcohol dependence and opiate dependence. (E = true)
--drstitch 22:47, 7 Nov 2007 (EST)
Which one of the following statements concerning Tramadol is FALSE?

A. It has an active metabolite
B. It inhibits serotonin and noradrenaline reuptake
C. It is LESS likely (at normal doses) to cause respiratory depression than other opioid agonists
D. It is metabolised in the liver and excreted in the kidneys
E. It structurally resembles codeine
A. It has an active metabolite TRUE, THEREFORE NOT THE ANSWER = O desmethyltramadol
B. It inhibits serotonin and noradrenaline reuptake TRUE AND THEREFORE NOT THE ANSWER [2]
C. It is less likely to (at normal doses) to cause respiratory depression than other opioid agonists TRUE AND THEREFORE NOT THE ANSWER. APMS third edition 2010 page 63.
D. It is metabolised in the liver and excreted in the kidneys TRUE AND THEREFORE NOT THE ANSWER [3]
E. It structurally resembles codeine FALSE AND THEREFORE THE ANSWER.
With regard to non-depolarising muscle relaxants:
A. NDMR actions are increased with hyperkalemia
B. Vecuronium exclusively (or primarily) undergoes renal elimination
C. Mivacurium half-life 10 minutes
D. Cisatracurium is metabolised to laudanosine by the liver
E. May require larger doses if treated with phenytoin and theophylline
A - False. (Hypokalaemia prolongs action, not hyperkalaemia)
B - False. Vecuronium undergoes elimination also by the liver (ie non-exclusive elimination)
C - False. (about 2 min)
D - False. Cisatracurium undergoes spontaneous metabolism to produce laudanosine, not organ dependent.
E - True. See CEACP article for confirmation
A ‘sleep dose’ of thiopentone in a healthy adult is likely to result in:
A. A fall in peripheral resistance followed by a rise in cardiac output
B. A fall in cardiac output followed by a rise in peripheral resistance
C. A fall in peripheral vascular resistance followed by a fall in cardiac output
D. A rise in peripheral vascular resistance followed by a fall in cardiac output
E. A fall in cardiac output followed by a fall in peripheral vascular resistance
A. A fall in peripheral resistance followed by a rise in cardiac output - false: CO falls
B. A fall in cardiac output followed by a rise in peripheral resistance
C. A fall in peripheral vascular resistance followed by a fall in cardiac output - probably most correct
D. A rise in peripheral vascular resistance followed by a fall in cardiac output
E. A fall in cardiac output followed by a fall in peripheral vascular resistance
Expected adverse drug effects in a geriatric population receiving a high dose
of a selective serotonin reuptake inhibitor for depression would include all
of the following EXCEPT

A. hyponatraemia caused by inappropriate secretion of ADH
B. impairment of platelet aggregation caused by depletion of 5HT (serotonin) stores
C. withdrawal symptoms characterised by anxiety, agitation and increased sweating
D. sedation, dry mouth, orthostatic hypotension and cardiac conduction defects
E. gastro-intestinal effects (nausea, vomiting, diarrhoea)
A. hyponatraemia caused by inappropriate secretion of ADH
B. impairment of platelet aggregation caused by depletion of 5HT (serotonin) stores
C. withdrawal symptoms characterised by anxiety, agitation and increased sweating
D. sedation, dry mouth, orthostatic hypotension and cardiac conduction defects - false and answer to choose: "The SSRIs do NOT cause sedation, dry mouth, orthostatic hypotension and cardiac conduction defects, all of which are seen with the tricyclics." (Selective serotonin reuptake inhibitors Pharmacology and clinical implications in anaesthesia and critical care medicine in Anaesthesia Volume 52, Issue 10, Date: October 1997, Pages: 982-988)
E. gastro-intestinal effects (nausea, vomiting, diarrhoea)
In patients with renal failure each of the following drugs has prolonged
clearance or has active metabolites with prolonged clearance EXCEPT

A. aspirin
B. buprenorphine
C. codeine
D. pethidine
E. tramadol
Buprenorphine
Fondaparinux Sodium (Arixtra)

A. activates platelet
B. cross reacts with sera from patients with heparin induced thrombocytopaenia
C. has a mechanism of action that is antithrombin (ATIII) dependent
D. is associated with thrombocytopaenia
E. can be safely used in patients with severe renal failure
C. has a mechanism of action that is antithrombin (ATIII) dependent - true: "Fondaparinux is a synthetic and specific inhibitor of activated factor X (Xa) with no animal sourced components. The antithrombotic activity of fondaparinux is the result of antithrombin III (ATIII) mediated selective inhibition of factor Xa. By binding selectively to ATIII, fondaparinux potentiates (about 300 times) the innate neutralisation of factor Xa by antithrombin. Neutralisation of factor Xa interrupts the blood coagulation cascade and inhibits both thrombin formation and thrombus development." (MIMs Online)
Regarding chemotherapy agents,
A. azathioprrne is a cholinesterase inhibitor and may interact with suxamethonium
B. bleomycin may have an idiosyncratic (i.e. not dose related) association with progressive respiratory fibrosis
C. cyclophosphamide has NO known interactions with neuromuscular blocking agents
D. high cumulative doses of doxorubicin are associated with cardiomyopathy
E. nonsteroidal anti-inflammatory drugs (NSAIDs) do NOT interact with methotrexate
B
Which of the following is NOT associated with serotonin syndrome
A. phenelzine
B. pethidine
C. ondansetron
D. chlorpromazine
E. sumatriptan
D
An adult male patient requires general anaesthesia. He admits to long-standing
substance abuse with central nervous system (CNS) stimulants. Compared with a
patient who is not a substance abuser, he is likely to require an
A. increased dose of induction agent and increased dose of opioid
B. increased dose of induction agent and reduced dose of opioid
C. increased dose of induction agent and unchanged dose of opioid
D. unchanged dose of induction agent and increased dose of opioid
E. unchanged dose of induction agent and unchanged dose of opioid
?B


"Acute amphetamine use dramatically increases anesthetic requirement(9,10) and has been implicated in a case of severe intraoperative intracranial hypertension (2) withoutpostoperative sequelae. Chronic amphetamine use results in amarkedly diminished anesthetic requirement(8), which is thought to result from catecholamine depletion in the CNS, and has been implicated in a case of cardiac arrest during general anesthesia. In a case report, Samuels et al. (1) speculated that chronic catecholamine depletion led to an inability to respond to the stress of anesthetic induction, specifically, the increase in venouscapacitance and the resulting decrease in preload that resulted from the induction dose of thiopental."
The plasma half-life of low molecular weight heparin is
A. Increased in conditions with raised plasma proteins
B. 2 to 4 times that of unfractionated heparin
C. Much less predictable than that of unfractionated heparin
D. Dependent upon a saturatable mechanism for clearance
E. Longer than unfractionated heparin because of a higher affinity for plasma proteins
B - True

The pharmacokinetics of enoxaparin (Clexane) and unfractionated heparin were compared by crossover study in healthy volunteers, using three different assays. After intravenous administration, unfractionated heparin was cleared with a half-life of 35 min, irrespective of assay methods. However, the concentration of enoxaparin, measured by competitive binding assay, declined with the longer half-life of 60 min, and its anti-Factor IIa and anti-Factor Xa activities had half-lives of 40 and 275 min, respectively....Comparison of the pharmacokinetics of enoxaparin (Clexane) and unfractionated heparin; Dawes J. (Heart Research Institute, NSW, Australia); Acta Chir Scand Suppl. 1990;556:68-74.
Intra-ocular pressure is increased by
A. head-up
B. hypothermia
C. metabolic acidosis
D. miosis
E. respiratory acidosis
?B
A morbidly obese 140kg, 40-year-old male is scheduled for cholecystectomy. He has no history of
cardiac disease. His ideal body weight is 70kg. Compared to his resting cardiac output at ideal
body weight, his resting cardiac output at his weight of 140 kg would be
A. decreased by 20% or more
B. decreased by 10%
C. unchanged
D. increased by 10%
E. increased by 20% or more
E?
In a clinical trial, 3 out of 10 patients develop a complication in the control group,
and 1 of 10 patients develops the complication in the treated group. To assess whether
this is a statistically significant difference the most appropriate statistical test to
use would be the

A. Chi-square Test
B. Chi-square Test with Yates correction
C. Student's t-test
D. Fisher’s Exact Test
E. Mann-Whitney Test
D
http://ceaccp.oxfordjournals.org/cgi/reprint/7/5/167
In a trial, 75 patients with an uncommon, newly described complication and 50 matched
patients without this complication are selected for comparison of their exposure to a
new drug. The results show

Complication present Complication absent
Exposed to new drug 50 25
NOT exposed 25 25

From these data

A. the relative risk of this complication with drug exposure CANNOT be determined
B. the odds ratio of this complication with drug exposure CANNOT be determined
C. the relative risk of this complication with drug exposure is 2
D. the odds ratio of this complication with drug exposure is 1.33 (recurring)
E. none of the above
A. the relative risk of this complication with drug exposure CANNOT be determined - true:

"Because accurate information concerning all patients at risk in a retrospective case-control study is not available (because sample size is set by the researcher), incidence rate and risk cannot be accurately determined, and the odds ratio is used as the estimate of the risk ratio" (Myles and gin p.74)
Forty patients are randomly dived into two groups - one to receive induction agent A and another
to receive induction agent B. The next day they are asked to rate their anaesthetic experience on
a scale of 1 (very bad) to 5 (very good). The most appropriate test to compare the anaesthetic
experience of the two groups is the
A. unpaired t-test
B. Mann-Whitney test
C. Chi-square test
D. Kruskal-Wallis test
E. paired t-test
B. Mann-Whitney test - true: The scale is ordinal. "If there is a natural order among categories, so that there is a relative value among them... then the data can be considered ordinal data... Ordinal data are... a type of categorical data." (Myles and Gin p 2-3). There are two independent groups and the equivalent of the unpaired t-test for non-parametric data is the Mann-Whitney U test. "Mann-Whitney U Test (identical to the Wilcoxon rank sum) is a non-parametric equivalent to the unpaired Student's t-test" (Myles and Gin p.63) "The Mann-Whitney U test is the recommended test to use when comparing two groups that have data measured on an ordinal scale. However, if the data represent a variable that is, in effect, a continuous quantity, then a t-test may be used if the data are normally distributed. This is more likely with large samples (say n>100)." (p. 64)
Publication Bias is that
A. researchers with a strong track record are more likely to get research published
B. studies with positive results are more likely to be published
C. studies with negative results are more likely to be published
D. studies on important clinical questions are more likely to be published
E. the prestige of the journal will affect readers' perception of the quality of the study
"Publication bias arises from the tendency for researchers and editors to handle experimental results that are positive (they found something) differently from results that are negative (found that something did not happen) or inconclusive"
FALSE statements regarding natural latex rubber allergy include

A. sensitivity of skin prick testing is greater than that of specific IgE antibody detection (RAST)
B. signs of Type I hypersensitivity are usually immediate
C. latex antibodies fall in time in a latex free environment
D. the risk factors of frequent exposure and atopy are additive
E. theatre latex aerogens are lowest in the morning
B

FALSE statements regarding natural latex rubber allergy include

A. sensitivity of skin prick testing is greater than that of specific IgE antibody detection (RAST)- true:
"RAST tests are highly specific, but their sensitivity is low... (while) the skin-prick test was very sensitive and specific" (Hepner and Castells, Latex allergy: an update from ANESTH ANALG 2003;96:1219–29)
B. signs of Type I hypersensitivity are usually immediate - false and the answer to choose:
"Latex proteins are absorbed slowly when the exposure is airborne, and symptoms usually develop approximately 30 min after exposure." (Hepner and Castells, Latex allergy: an update from ANESTH ANALG 2003;96:1219–29)
C. latex antibodies fall in time in a latex free environment - true:
"A recent follow-up study (72) on latex allergy in health care workers demonstrated that although the skin-prick test remained positive 2 yr after latex avoidance, latex specific IgE levels decreased in most patients. In addition, allergic symptoms decreased in most patients." (Hepner and Castells, Latex allergy: an update from ANESTH ANALG 2003;96:1219–29)
D. the risk factors of frequent exposure and atopy are additive - true:
"The prevalence of latex sensitization is less than 1% in a nonatopic normal population, whereas the prevalence in health care workers ranges from 3%–12% (37). The incidence of latex sensitization, as measured by IgE levels, in ambulatory surgical patients at one institution was as frequent as 6.7% (1). Although sensitization does not always lead to an anaphylactic reaction, continued exposure to latex will increase the possibility of such a reaction. A health care worker who is atopic is at increased risk, and the risk is increased if they had a previous surgery (40). Anesthesiologists have a 12.5% and 2.4% prevalence of latex sensitization and allergy, respectively (2). Adult anesthesiologists change gloves more often than pediatric anesthesiologists and have been demonstrated to have an increased prevalence of latex sensitization (41). There is no increased risk of latex allergy with age, race, or sex, and exposure is the single most significant factor associated with latex allergy (2,42)." (Hepner and Castells, Latex allergy: an update from ANESTH ANALG 2003;96:1219–29)
E. theatre latex aerogens are lowest in the morning - true:
"The American Society of Anesthesiologists Task Force of Latex Sensitivity recommends that patients who are latex allergic have a surgical procedure performed as the first case in the morning, when the levels of latex aeroallergens are the smallest" (Hepner and Castells, Latex allergy: an update from ANESTH ANALG 2003;96:1219–29)
Cryoprecipitate contains each of the following clotting factors EXCEPT

A. factor VIII
B. factor IX
C. factor XIII
D. fibrinogen
E. von Willebrand factor
Cryoprecipitate contains each of the following clotting factors EXCEPT

A. factor VIII - false: see above
B. factor IX - most true: by deduction
C. factor XIII - false: see above
D. fibrinogen - false: see above
E. von Willebrand factor - false: see above
A fifty-five-year-old man on antihypertensive medication, including an ACE inhibitor, has a total
knee replacement. Red cell transfusion is begun in recovery through a leukocyte reduction filter
after brisk bleeding into his drains. A recognised complication of the use of this filter in
this situation is

A. air embolism
B. clotting factor depletion
C. haemolysis
D. increased risk of postoperative infection
E. severe hypotension
A fifty-five-year-old man on antihypertensive medication, including an ACE inhibitor, has a total knee replacement. Red cell transfusion is begun in recovery through a leukocyte reduction filter after brisk bleeding into his drains. A recognised complication of the use of this filter in this situation is

A. air embolism
B. clotting factor depletion
C. haemolysis
D. increased risk of postoperative infection
E. severe hypotension - true: "Profound hypotension has been reported in patients taking angiotension-converting-enzyme (ACE) inhibitors and receiving pretransfusion leukocytereduced blood products—platelets in particular (96). Presumably, ACE inhibitors decrease bradykinin degradation thereby prolonging its intravascular half-life." (Sharma et al, Leukocyte-reduced blood transfusions: perioperative indications, adverse effects, and cost analysis, in Anesth Analg. 2000 Jun;90(6):1315-23)
The administration of blood contaminated with Yersinia enterocolitica will typically
produce symptoms of infection

A. during the transfusion
B. within 6 hours
C. within 12 hours
D. within 24 hours
E. within a week
A. during the transfusion - true: "The onset of clinical symptoms typically occurs acutely during transfusion, with a mortality rate of 60% and a median time to death of only 25 hrs (27)." (Goodnough, Risks of blood transfusion, in Crit Care Med 2003 Vol. 31, No. 12 (Suppl.))
TRALI after administration of blood products occurs
A. immediately
B. within 4 hours
C. within 8 hours
D. within 24 hours
E. within 48 hours
...The TRALI consensus conference and the US National Heart, Lung and Blood Institute defined TRALI as a form of acute lung injury meeting the criteria proposed by the AECC that is temporally and mechanistically related to transfusion of blood or blood components [6, 7, 15]. For a diagnosis of TRALI to be made there must be no pre-existing ALI before transfusion, the onset of signs and symptoms must occur during or within 6 h of transfusion and there must be no temporal relationship to an alternative risk factor for ALI...
The most frequently reported cause of mortality associated with transfusion of blood and blood products is
A. anaphylaxis
B. bacterial sepsis
C. haemolytic reaction
D. transfusion associated graft versus host disease
E. Transfusion Related Acute Lung Injury (TRALI)
E
In a patient requiring FFP where the patient’s blood group is unknown, it is ideal to give FFP of group
A. A
B. B
C. AB
D. O
E. Blood group of FFP in this situation does not matter
C. AB - CORRECT: "Group AB FFP can be used in an emergency if the patient’s ABO blood group is unknown, but is likely to be in short supply... With regard to ABO blood groups, the first choice of FFP is that of the same ABO group as the patient. If this is not available, FFP of a different ABO group is acceptable so long as it has been shown not to possess anti-A or anti-B activity above a limit designed to detect ‘high titres’. FFP of group O should only be given to O recipients" (British Committee for Standards in Haematology, Blood Transfusion Task Force, Guidelines for the use of fresh-frozen plasma, cryoprecipitate and cryosupernatant, in British Journal of Haematology 2004:126, 11–28)
When providing general anaesthesia to a patient with a history of asthma

A. thiopentone should not be used as it may cause bronchospasm
B. intravenous and topical tracheal lignocaine are equally effective in preventing bronchial
hyperreactivity
C. ketamine provides little benefit in a patient with active wheezing
D. induction with propofol is effective in reducing the incidence of wheezing following intubation
E. isoflurane is as effective a bronchodilator as halothane when given in MAC equivalent doses
When providing general anaesthesia to a patient with a history of asthma

A. thiopentone should not be used as it may cause bronchospasm - false:
"When general anesthesia is selected, induction of anesthesia is most often accomplished with an intravenous induction drug. The incidence of wheezing is higher in asthmatic patients receiving thiopental for induction than in those given propofol. Thiopental itself does not cause bronchospasm, but it may inadequately suppress upper airway reflexes so airway instrumentation may trigger bronchospasm." (From Stoelting's Anesthesia and Co-existing Diease 5E Ch 9 p.167)
B. intravenous and topical tracheal lignocaine are equally effective in preventing bronchial hyperreactivity - unsure but probably incorrect:
"An alternative method to suppress airway reflexes prior to intubation is the intravenous or intratracheal injection of lidocaine 1 to 3 minutes before tracheal intubation" (Stoelting's Anesthesia and Co-existing Disease, 5E, Ch 9, p. 167)
C. ketamine provides little benefit in a patient with active wheezing - false:
"Ketamine may produce smooth muscle relaxation and contribute to decreased airway resistance, especially in patients who are actively wheezing." (Stoelting's Anesthesia and Co-existing Disease, 5E, Ch 9, p. 167)
D. induction with propofol is effective in reducing the incidence of wheezing following intubation - true:
"When general anesthesia is selected, induction of anesthesia is most often accomplished with an intravenous induction drug. The incidence of wheezing is higher in asthmatic patients receiving thiopental for induction than in those given propofol... The mechanism of propofol's relative bronchodilating effect is unknown." (Stoelting's Anesthesia and Co-existing Disease, 5E, Ch 9, p. 167)
E. isoflurane is as effective a bronchodilator as halothane when given in MAC equivalent doses - false:
"The lesser pungency of halothane and sevoflurane (compared with isoflurane and desflurane) may make coughing, which can trigger bronchospasm, less likely" (Stoelting's Anesthesia and Co-existing Disease, 5E, Ch 9, p. 167)
AC22e [Jul06] [Apr07] Detrimental post-operative effects of hypothermia following general
anaesthesia in the elderly include each of the following EXCEPT:
A. delayed emergence
B. hypercoagulability
C. increased body metabolism
D. reduced elimination of anaesthetic agents
E. shivering
Detrimental post-operative effects of hypothermia following general anaesthesia in the elderly include each of the following EXCEPT:

A. delayed emergence - true: Look up R. Lenhardt, E. Marker and V. Goll et al., Mild intraoperative hypothermia prolongs postoperative recovery, Anesthesiology 87 (1997), pp. 1318–1323.
B. hypercoagulability - false and answer to choose: "A clinical bleeding diathesis is associated with both deliberate and inadvertent hypothermia." (Reynolds et al, Perioperative complications of hypothermia, in Best Practice & Research Clinical Anaesthesiology Volume 22, Issue 4, December 2008, Pages 645-657)
C. increased body metabolism - true: "Another potentially complication related to shivering is the increase in metabolic demands." (Reynolds et al, Perioperative complications of hypothermia, in Best Practice & Research Clinical Anaesthesiology Volume 22, Issue 4, December 2008, Pages 645-657)
D. reduced elimination of anaesthetic agents - true: "Propofol, a commonly used anaesthetic is paradoxically affected by temperature; a 3 °C decrease in core temperature results in an approximate 30% increase in plasma concentration of propofol. Interestingly, mild hypothermia does not alter hepatic blood flow1, or propofol requirement during craniotomy surgery.2 It is postulated that the increase in propofol plasma concentration is due to a reduced inter-compartmental clearances between the central and shallow compartments. Fentanyl has also been shown to have a relationship with hypothermia; a 5%/°C increase in steady-state plasma concentration of fentanyl can be observed." (Reynolds et al, Perioperative complications of hypothermia, in Best Practice & Research Clinical Anaesthesiology Volume 22, Issue 4, December 2008, Pages 645-657)
E. shivering - maybe true, maybe false: "Shivering is an important complication of hypothermia... Shivering however, is rare in elderly due to impaired thermoregulatory control and thus the potential increase in oxygen consumption is generally absent in this population." (Reynolds et al, Perioperative complications of hypothermia, in Best Practice & Research Clinical Anaesthesiology Volume 22, Issue 4, December 2008, Pages 645-657)
Effects of hypothermia at 25C:
A. Hyperglycaemia
B. Oxygen consumption is decreased by greater than 50%
C. Cerebral blood flow is decreased by 25%
D. Coronary blood flow is decreased
E. Increased coagulability of blood
F. Decreased viscosity
G. Increased BSL
H. ? Decrease in K+
Effects of hypothermia at 25C:

A. Hyperglycaemia - most true: "At 32 degrees, hyperglycaemia is common due to decreased insulin release and lowered peripheral utilisation" (Morrison, Hypothermia in the elderly, in International Anesthesiology Clinics, Vol 26 No 2 1988)
B. Oxygen consumption is decreased by greater than 50% - false; From "The ugly" table, "increased oxygen consumption by as much as 400% to 700%" (Morrison, Hypothermia in the elderly, in International Anesthesiology Clinics, Vol 26 No 2 1988)
C. Cerebral blood flow is decreased by 25% - ?
D. Coronary blood flow is decreased - ?
E. Increased coagulability of blood - partly true: "Intravascular rouleaux formation and red blood cell aggregation occur in peripheral microcirculation at 28C, and microvascular sludging may become a problem. Coagulation changes also occur during progressive cooling as platelets, coagulation cascade proteins, protein clearance by the liver, and coagulation inhibitors are affected. Progressive thrombocytopenia occurs as platelets are sequestered in the portal circulation and liver. Hepatic function is directly depressed during cooling, and coagulation degradation products are cleared less repidly. This effect may lead to regional hypercoagulable states and, in extreme cases, DIC." Morrison, Hypothermia in the elderly, in International Anesthesiology Clinics, Vol 26 No 2 1988)
F. Decreased viscosity - false: "Blood viscosity increases 173% in dogs cooled from 37 degrees to 25 degrees Celsius" (Morrison, Hypothermia in the elderly, in International Anesthesiology Clinics, Vol 26 No 2 1988)
G. Increased BSL - true: see answer to A
H. ? Decrease in K+ - ?
The most commonly reported cause of awareness during general anaesthesia for a non-obstetric
procedure is

A. inadequate intra-operative opiate analgesia
B. equipment failure
C. the use of total intravenous anaesthesia
D. faulty anaesthetic technique
E. use of a laryngeal mask airway
D
The clinical sign which has the highest correlation with awareness in a spontaneously
breathing patient under general anaesthesia is

A. patient movement in response to a stimulus
B. an increase in blood pressure and heart rate
C. increased lacrimation and sweating
D. pupillary dilation
E. an increase in respiratory rate
A

The most common cause of awareness is faulty anaesthetic technique (CEACCP Dec 2005)[1]
The most reliable sign of awareness is movement
The most important factor in reducing peri-operative morbidity in diabetic patients
undergoing peripheral vascular surgery is
A. tight control of blood sugar level in the peri-operative period
B. frequent blood sugar level estimations
C. use of regional rather than general anaesthesia
D. stabilisation of co-existing disease
E. the use of an insulin infusion rather than a subcutaneous sliding scale regimen
D
A healthy 20 year old patient undergoing nasal surgery under general anaesthesia has the nose packed with gauze
soaked in 0.5% phenylephrine and a submucosal injection of lignocaine with 1:100,000 adrenaline.
Over the next 10 minutes the blood pressure rises from 130/80 to 220/120 mmHg and the heart rate from 60 to 100
beats per minute. The LEAST appropriate management of this situation would be to
A. deepen anaesthesia with isoflurane
B. administer labetalol
C. watch and observe
D. administer glyceryl trinitrate
E. administer sodium nitroprusside
B (beta-blockers in general are bad here as negative inotropy in face of increased peripheral resistance leads to cardiac arrest)
During surgery for tracheostomy insertion surgical diathermy is being used at the tracheal
incision. You are ventilating with 100% oxygen. As the trachea is opened you notice a blue
flame shooting up from the incision. Your first action should be to

A. disconnect the breathing circuit from the endotracheal tube
B. douse the wound with saline
C. insert a tracheostomy tube
D. remove the endotracheal tube
E. turn off oxygen and ventilate with air
A
Appropriate practice when performing central neuro-axial block should include all of the
following EXCEPT

A. application of an alcohol based antiseptic at least twice
B. application of an alcohol based antiseptic that is allowed to dry
C. a surgical hand wash
D. disinfection of the neck of ampoules being cracked open by an assistant
E. wearing a new facemask
D
Which of the following contributes the LEAST to motor neuropathy following the lithotomy
position?

A. age
B. BMI (body mass index) >25
C. diabetes
D. peripheral vascular disease
E. smoking
D. BMI >25 - correct answer: "Of the patient-related characteristics, low BMI was a powerful univariate risk factor for motor neuropathy" (Warner, et al., Lower-extremity Motor Neuropathy Associated with Surgery Performed on Patients in a Lithotomy position in Anesthesiology 81:6-12, 1984) Perhaps this is due to less self-padding?
Regarding Epidural Abcess - which is WRONG
A. Diagnosis is DEPENDENT on triad of back pain, fever, and paralysis
B. Occurs at a rate of 1:1000-3000 (OR 1:2000 - 1:5000)
C. Worse outcomes if advanced age
D. Usually gram positive cocci
E. Expectant management may be appropriate
A. Diagnosis is DEPENDENT on triad of back pain, fever, and paralysis - false and the ANSWER to CHOOSE: "The early signs and symptoms may be vague, the 'classic' triad of back pain, fever and variable neurological deficit occurred in only 13% of patients by the time of diagnosis, and contributed to diagnostic delay in 75%." (Grewal et al, Epidural Abscesses in BJA 2006 96(3):292-302)
AC154 [Apr07] Q128

In modern anaesthetic practice, mortality from myocardial infarction
after non cardiac surgery is

A. 2.5-5%
B. 5-10%
C. 10-15%
D. 20-30%
E. approximately 50%
C
A patient with severe COPD on home oxygen is having an excision of a submandibular
tumour under local anaesthesia. The best way to prevent fire in the operating room is:
A. seal the surgical site from the patients airway with adhesive drapes
B. use bipolar instead of monopolar diathermy
C. decr FIO2 to maintain sats 97%
D. use alcoholic chlorhex instead of iodine
E. add nitrous oxide to the inhaled gases to reduce the FiO2 and provide sedation
A patient with severe COPD on home oxygen is having an excision of a submandibular tumour under local anaesthesia. The best way to prevent fire in the operating room is:

A. seal the surgical site from the patients airway with adhesive drapes - potentially correct: "Oxygen is heavier than air, and can therefore accumulate under surgical drapes. This accumulation may be reduced by the use of ‘incise drapes’ that protect the wound from high oxygen concentrations and by tenting surgical drapes to dilute oxygen with room air." (Muchatuta and Sale, Fires and explosions, Anaesth&IC, 2007, 8:11) The answer will depend on what the examiners mean by adhesive drapes
B. use bipolar instead of monopolar diathermy - definitely correct: "The cutting mode of diathermy is more likely to ignite fuels than the coagulation mode, and fires are more likely with monopolar diathermy than bipolar" (Muchatuta and Sale, Fires and explosions, Anaesth&IC, 2007, 8:11)
C. decr FIO2 to maintain sats 97% - almost certainly incorrect: "Strategies to reduce the risks posed by high oxygen concentrations include (the) judicious use of oxygen (using the lowest oxygen concentration that provides acceptable haemoglobin oxygen saturations" (Muchatuta and Sale, Fires and explosions, Anaesth&IC, 2007, 8:11) The key here is acceptable haemoglobin oxygen saturations. "Long term continuous oxygen therapy should be considered for patients with stable chronic lung disease, particularly COPD, who have an arterial PO2 (PaO2) consistently less than or equal to 55 mm Hg when breathing air, at rest and awake. ...Flow rate should be set to maintain PaO2 > 60mmHg (8 kPa) (oxygen saturation level, measured by pulse oximetry [SpO2]> 90%) during waking rest." (McDonald et al, Adult domiciliary oxygen therapy. Position statement of the Thoracic Society of Australia and New Zealand, MJA 2005; 182: 621–626) Note that PO2=55mmHg is equivalent to SaO2 88%
D. use alcoholic chlorhex instead of iodine - definitely incorrect: "Alcohol-based antibacterial skin preparations are one of the more common causes of surgical fires since the withdrawal of flammable anaesthetic agents. They can pool on the body surface (especially umbilicus and suprasternal notch), be wicked into surgical drapes and produce flammable vapours that can accumulate beneath the drapes." (Muchatuta and Sale, Fires and explosions, Anaesth&IC, 2007, 8:11)
E. add nitrous oxide to the inhaled gases to reduce the FiO2 and provide sedation - definitely incorrect: "Nitrous oxide also supports combustion and is broken down to produce oxygen, nitrogen and heat." (Muchatuta and Sale, Fires and explosions, Anaesth&IC, 2007,
In the Neuroleptic malignant syndrome

A. there is a familial incidence
B. non-depolarizing muscle relaxants decrease the muscle rigidity
C. creatinine kinase levels are elevated following an episode
D. there is an association with malignant hyperpyrexia
E. hyperthermia does not always occur
In the Neuroleptic malignant syndrome

A. there is a familial incidence - false, but now true: "there is no familial pattern or evidence of inheritance in neuroleptic malignant syndrome" (Stoelting's A and CED Ch22)
"Familial clusters of NMS suggest a genetic predisposition to the disorder [30]. Genetic studies have shown that the presence of a specific allele of the dopamine D2 receptor gene is over represented in NMS patients [31]. This allele is associated with reduced density and function of dopamine receptors as well as decreased dopaminergic activity and metabolism." (Uptodate)
B. non-depolarizing muscle relaxants decrease the muscle rigidity - true:
"Many of the previous reports documented that succinylcholine may be safely used during ECT in a patient of NMS3, but some do not agree8, as even small dose of succinylcholine can produce side effects. Reports quote that mivacurium (0.15-0.25mg kg-1) and rapacuronium (0.6- 0.8 mg kg-1) may be safe alternatives to succinylcholine.4,9,10"(Alternative to Succinylcholine for ECT in Neuroleptic Malignant Syndrome Patients: Can Atracurium be Safe?, J Anaesth Clin Pharmacol 2008; 24(2): 229-230[1])
"A distinguishing feature is the ability of nondepolarizing muscle relaxants to produce flaccid paralysis in patients experiencing the neuroleptic malignant syndrome but not in those experienceing malignant hyperthermia." (Stoelting Pharm and Phys 3rd ed p. 372)
C. creatinine kinase levels are elevated following an episode - false: If this said creatine kinase, it would definitely be true.
"NMS and malignant hyperthermia have clinical features in common, including hyperthermia, rigidity, an elevated creatine kinase concentration and a mortality rate for both NMS and malignant hyperthermia of 10–30%" Note that it is CREATINE KINASE not CREATININE KINASE (BJA (2000) 85; 129-35)
D. there is an association with malignant hyperpyrexia - false: "the association between NMS and other potentially fatal syndromes, such as malignant hyperthermia, is unclear" (BJA (2000) 85; 129-35)
E. hyperthermia does not always occur - false: "Three major symptoms indicate a high probability of the presence of NMS: hyperthermia, rigidity and an elevated creatine phosphokinase concentration, reflecting rhabdomyolysis34 (Table 1). In the absence of these criteria, the diagnosis of NMS should be questioned, since other symptoms of the disorder may be seen in patients taking neuroleptics without having NMS. Elevated temperature (38.5°C) in the absence of other systemic illness is observed in most patients." (BJA (2000) 85; 129-35)
The diagnosis of neuroleptic malignant syndrome requires the presence of:

A. Diaphoresis
B. elevated plasma creatinine kinase
C. hypertension
D. muscle rigidity
E. tachycardia
A. Diaphoresis
B. elevated plasma creatinine kinase
C. hypertension
D. muscle rigidity - best answer
E. tachycardia

In the DSM-IV, there are criteria for diagnosis as follows (from Crit Care Med 2010 Vol. 38, No. 6 (Suppl) S244-52)

A - Development of severe muscle rigidity and elevated temperature associated with the use of neuroleptic medication
B - Two or more of; Diaphoresis, dysphagia, tremor, incontinence, changes in level of consciousness ranging from confusion to coma, mutism, tachycardia, elevated or labile blood pressure, leukocytosis, laboratory evidence of muscle injury (ie, elevated creatine phosphokinase)
C - The symptoms in criteria A and B are not due to another substance (eg, phencyclidine) or a neurologic or other general medical condition (eg, viral encephalitis)
D - The symptoms in criteria A and B are not better accounted for by a mental disorder (eg mood disorder with catatonic features)
In a patient with myasthenia gravis undergoing a laparotomy for large bowel obstruction, the
need for post-operative ventilation is significantly increased by a

A. daily dose of pyridostigmine > 180mg
B. known history of resistance to suxamethonium
C. known history of sensitivity to non-depolarising muscle relaxants
D. past history of prednisolone treatment > 10 mg.day-1
E. recent history of dysphagia
A. daily dose of pyridostigmine > 180mg - false: "Criteria that correlate with the need for mechanical ventilation during the postoperative period following transsternal thymectomy include disease duration longer than 6 years, the presence of COPD unrelated to MG, a daily dose of pyridostigmine higher than 750mg, and a VC less than 2.9L" (Stoelting's Anesthesia and Co-Existing Disease, 5E, p.452)
B. known history of resistance to suxamethonium
C. known history of sensitivity to non-depolarising muscle relaxants
D. past history of prednisolone treatment > 10 mg.day-1
E. recent history of dysphagia - true: " Any degree of bulbar palsy is predictive of the need for both intra- and post-operative ventilation" OHA(pg246)
In patients with myasthenia gravis, features which increase the risk of requiring prolonged
post-operative ventilation, include each of the following EXCEPT

A. a high dose of pyridostigmine preoperatively
B. a long history of the disease
C. high sensitiviey to neuromuscular blocking agents
D. history of a previous respiratory crisis
E. presence of a bulbar palsy
C

According to Oxford Handbook:

Preoperative predictors of postoperative need for ventilation:

duration of disease of greater than 6 years
history of coexisting chronic resp disease
dose requirements of pyridostigmine > 750 mg/day less than 48 H prior to surgery
preoperative VC < 2.9L
Each of the following is effective in the treatment of pain from acute herpes zoster EXCEPT

A. acyclovir
B. amitriptyline
C. carbamazepine
D. corticosteroids
E. topical aspirin
A. acyclovir - true: "Antiviral agents started within 72 hours of onset of the herpes zoster rash accelerate the resolution of acute pain (U) (Level I), but do not reduce the incidence of postherpetic neuralgia"
B. amitriptyline - true: "Amitriptyline (used in low doses for 90 days from onset of the herpes zoster rash) reduces the incidence of postherpetic neuralgia"
C. carbamazepine - false: Looks like carbamazepine is good for trigeminal neuralgia but not HZ in other distributions
D. corticosteroids - true: "Prednisolone added to acyclovir during HZ resulted in a modest reduction in pain intensity and improved the rate of skin lesion healing for up to 14 days, with no effect on the overall recovery rate at 3 weeks (Wood et al, 1994 Level II). Prednisolone, either as monotherapy or in combination with acyclovir, increased the likelihood of being ‘pain-free’ at 1 month by a factor of 2.3 (95% CI: 1.4 to 3.5), however there was no difference in the rate of skin healing, compared with placebo"
E. topical aspirin - true: "Topical aspirin, topical lignocaine patch or oxycodone controlled release, provide analgesia in herpes zoster"
In the treatment of phantom limb pain

A. calcitonin infusion is NOT effective
B. gabapentin reduces the pain
C. intravenous lignocaine reduces the pain
D. ketamine provides long-term pain relief
E. opiates are NOT effective
B
A strategy shown to reduce the incidence of severe phantom limb pain
is the use of
A. continuous regional blockade using nerve sheath catheters
B. patient controlled analgesia with opioids post-op
C. perioperative ketamine
D. perioperative NSAIDs
E. spinal anaesthesia for the amputation
A. continuous regional blockade using nerve sheath catheters - false: "Infusions of local anaesthetics via peripheral nerve sheath catheters, usually inserted by the surgeon at the time of amputation, are a safe method of providing excellent analgesia in the immediate postoperative period (Pinzur et al, 1996 Level II; Lambert et al, 2001 Level II). However, they are of no proven benefit in preventing phantom pain or stump pain (Halbert et al, 2002 Level I)."
B. patient controlled analgesia with opioids post-op
C. perioperative ketamine - true: "A small observational study found that while the overall incidence of long-term phantom limb pain was similar in patients given ketamine (bolus dose followed by an infusion, started prior to skin incision and continued for 72 hours postoperatively) compared with no ketamine, the incidence of severe phantom limb pain was reduced in the ketamine group (Dertwinkel et al, 2002 Level III-3). Another study looking at the effects of ketamine reported a numerical but not statistically significant difference in the incidence of phantom limb pain at 6 months after amputation (47% in the ketamine group and 71% in the control group) (Hayes et al, 2004 Level II)."
D. perioperative NSAIDs
E. spinal anaesthesia for the amputation
When compared with intra-muscular or subcutaneous opioid regimens, patient controlled analgesia (PCA} with opioids
A. is equally preferred by patients
B. provides better analgesia
C. results in less opioid-related adverse effects
D. results in lower opioid consumption
E. results in shorter hospital stay
B

APM Update 2007 (via college
website) Intravenous opioid PCA provides better analgesia than conventional parenteral opioid regimens (Level I [Cochrane Review]). Patient preference and satisfaction for intravenous PCA are higher when compared with conventional regimens (Level I [Cochrane Review]). Opioid administration by IV PCA leads to slightly higher opioid consumption. There is no difference in the incidence of nausea and vomiting compared with conventional regimens but the incidence of pruritus is increased. There is no difference in length of hospital stay (Level I [Cochrane Review]).
A 60-year-old diabetic has had a below knee amputation for an ischaemic leg. He has neuropathic pain being managed with oxycodoiie 40 mg bd and paracetamol 1 g
qid. He is also on omeprazole 20 mg daily for reflux. You decide to commence gabapentin. Before deciding on a dosage regimen and commencing therapy it is most
important that you
A. cease his omeprazole
B. check his hepatic transaminase level
C. check his renal function
D. check his QT interval on a resting EGG
E. reduce his oxycodone dose
C
Major complications of mediastinoscopy include all of the following EXCEPT

A. compression of the great vessels
B. air embolism
C. pneumothorax
D. major haemorrhage
E. phrenic nerve damage
Major complications of mediastinoscopy include all of the following EXCEPT

A. compression of the great vessels - maybe. Compression of major vessels is noted to be a reason for invasive blood pressure monitoring in the CEACCP article but is not listed in the table of complications, perhaps because it is a transient phenomenon rather than a complication per se.
B. air embolism - true: See Table 3 in (Continuing Education in Anaesthesia, Critical Care & Pain 2007 7(1):6-9[1])
C. pneumothorax - true
D. major haemorrhage - true
E. phrenic nerve damage - true
Following one-lung ventilation there is an increased risk of lung injury if plateau airway
pressure (during one-lung ventilation) exceeds

A. 20 cmH2o
B. 30 cmH2O
C. 40 cmH20
D. 50 cmH20
E. no relation to pressure
B
Following a left sided pneumonectomy, a left intercostal drain is placed and
connected to an underwater drainage system. In the postoperative period

A. A leakage of air is expected from the drain
B. The patient should be nursed in the right lateral decubitus position
C. The underwater seal drain should be left on continuous free drainage
D. The underwater seal drain should be left on continuous free drainage, and
connected to wall suction for 5 minutes every hour
E. The underwater seal drain should remain clamped and be released for a
short period every hour
E

"Following pneumonectomy, a single drain is placed which should be clamped and intermittently unclamped, this drain must not be on suction as cardiac herniation may occur."

Regarding caridac henriation:

"If the pericardium has been disrupted following pneumonectomy, herniation of the heart may occur through the defect. The patient should be immediately turned with the non-operative lung dependent. If the chest drain on the operative side had mistakenly put on suction, it
should be opened to air."
A patient with severe liver disease is scheduled to have a portacaval shunt for portal hypertension.
Recommended features of the anaesthetic management include

A. use of vecuronium as the relaxant of choice
B. care with the dose of suxamethonium because it may have a prolonged effect
C. avoidance of fentanyl as an analgesic
D. avoidance of all volatile anaesthetic agents including isoflurane
E. reduction of the induction dose of thiopentone
A. use of vecuruonium as the relaxant of choice
B. care with the dose of suxamethonium because it may have a prolonged effect - false: "The metabolism of succinylcholine may be slowed because of reduced pseudocholinesterase concentrations, but in practice this gives few problems" (Contin Educ Anaesth Crit Care Pain 2010 10: 15-19)
C. avoidance of fentanyl as an analgesic
D. avoidance of all volatile anaesthetic agents including isoflurane
E. reduction of the induction dose of thiopentone - true: "The dose of thiopental should be reduced because a reduction in plasma proteins results in an increased unbound fraction of drug; the distribution half-life and consequently the duration of action are also prolonged" (Contin Educ Anaesth Crit Care Pain 2010 10: 15-19)
Regarding extracorporeal shockwave lithotripsy (ESWL) for renal calculi in patients with a
permanent pacemaker in situ,

A. ESWL is contraindicated
B. ESWL pulses should be timed to coincide with the ST interval
C. rate modulation of the pacemaker should be deactivated during the ESWL procedure
D. there is a risk that the ESWL pulse will reprogramme the pacemaker
E. regional anaesthesia is relatively contraindicated
A. ESWL is contraindicated
B. ESWL pulses should be timed to coincide with the ST interval
C. rate modulation of the pacemaker should be deactivated during the ESWL procedure - true: "Lithotripsy may be used in patients with a pacemaker insitu, but the lithotriptor must be kept at least 6 inches away from the pacemaker. The lithotripsy pulses should be timed with the ECG and rate-modulation should be de-activated" (Diprose and Pierce, Anaesthesia for patients with pacemakers and similar devices, in CEACCP 2001(6):166)
D. there is a risk that the ESWL pulse will reprogramme the pacemaker
E. regional anaesthesia is relatively contraindicated
The primary duty of an expert witness called by the defence is to the

A. Court
B. Defence Counsel
C. defendent doctor
D. medical profession
E. patient taking legal action
A is best answer

Be truthful
Be impartial
Must NOT have financial or other interest in the outcome of the case
Must NOT adopt a position of advocacy
Must NOT favour plaintiff or defendant
During elective major vascular surgery the best way to reduce the risk of acute renal failure is
to maintain a normal

A. central venous pressure
B. mean arterial blood pressure
C. renal blood flow
D. systemic vascular resistance
E. urine output
C
In the eldery patient
A. sedation does not negate the benefits of regional anaesthesia with respect to postoperative cognitive dysfunction
B. the clearance of suxamethonium is reduced
C. shivering may be less effective in restoring body temperature, than in younger adults
D. the levels of vasopressin and atrial natiuretic peptide are similar to those in younger patients
E. the uptake of sevoflurane is similar to that in younger adults, because of its low blood gas solubility
C. True "elderly or debilitated patients shiver less effectively than younger patients" Perioperative hypothermia in the high-risk surgical patient. Best Practice & Research Clinical Anaesthesiology, Volume 17, Issue 4, Pages 485-498 K. Leslie, D. Sessler
Patient having THR; BP normal; under GA with sevo/N2O/fentanyl, immediately before surgeon cementing, best thing to do:

A. Induce hypotension

B. Raise BP with vasopressors

C. Turn off nitrous

D. Give steroids

E. Give heparin 5000u
?
You have anaesthetised a patient with a partially obstructing right main bronchus tumour.
You are using Heliox (28% O2:72% He) and sevoflurane. You secure the airway with
an endotracheal tube and continue with the same gas mixture. After 20 minutes the ETCO2 is 28mmHg.
The capnogram waveform is normal. This probably means the patient is
A. Appropriately ventilated with a low cardiac output
B. Appropriately ventilated with a normal cardiac output
C. hypothermic (34C)
D. Hyperventilating
E. Hypoventilating
B correct

Capnography is considered essential in the management of mechanically-ventilated patients. Helium, as an adjunct to mechanical ventilation, is the subject of renewed interest and used increasingly. However, helium affects the performance of infrared capnometry. We constructed a simple device to generate variable mixtures of helium, oxygen and carbon dioxide within the normal physiological range, and tested the performance of two side-stream and one in-line capnographs. We found that addition of helium to the gas mixture caused all three capnographs to underestimate the concentration of carbon dioxide. The underestimation increased as the proportion of helium increased. The maximum underestimation (30%) occurred in a 79:21 helium ⁄ oxygen mixture.

Also, the answer is conveniently

40mmHg x 70% = 28mmHg

which is exactly rhe same as Answer B.
The diagram below respresents a transgastric short axis view of the heart obtained using
transoesophageal echocardiography. The structure labelled A is the
A. anterolaterlal papillary muscle
B. non-coronary leaflet of the aortic valve
C. posterior mitral valve commissure
D. posterolateral papillary muscle
E. posteromedial papillary muscle

[Image showing arrow pointing to a structure encroaching on lumen of LV from posterior wall]
[Note there was a handy key to the left of the image to show anterior vs inferior, left vs right]

http://www.jeffmott.net/ANZCA/EM35c.jpg
E
Systemic vascular resistance index (SVRI) is calculated from

A. systemic vascular resistance multiplied by body surface area
B. systemic vascular resistance divided by body surface area
C. mean aortic and central venous pressure difference divided by cardiac output
D. cardiac index divided by the mean aortic and central venous pressure difference
E. none of the above
A

We all know that SVR = ((MAP-CVP)*80)/CO

We also all know that CI = CO/BSA

SVRI is calculated by substituting CI for CO in the equation for SVR, i.e. SVRI = ((MAP-CVP)*80)/CI (it is, go and look it up)

If we do some substituting we get

SVRI = ((MAP-CVP)*80)/(CO/BSA)
SVRI = ((MAP-CVP)*80)*(BSA/CO)
SVRI = (((MAP-CVP)*80)/CO)*BSA
SVRI = SVR * BSA
The risk of complications with insertion of a Swan-Ganz catheter is increased in all
of the following EXCEPT

A. coagulopathy
B. left bundle branch block
C. prosthetic right heart valve
D. recently inserted endocardial pacemaker leads
E. aortic stenosis
?B
Which statement about the use of Entropy depth of anaesthetic monitoring is true?

A. artefacts caused by cardiac pacemaker produce a high RE (Response Entropy) value
B. RE only assesses the EEG (electroencephalogram) range from 32-47Hz (high frequency)
C. SE (State Entropy) values range from 0 – 100
D. SE is a stable indicator of the effects of hypnotics on the cortex
E. the RE algorithm filters the EMG (electromyogram) component
A. artefacts caused by cardiac pacemaker produce a high RE (Response Entropy) value - false
B. RE only assesses the EEG (electroencephalogram) range from 32-47Hz (high frequency) - false: "The response entropy (RE) is computed over a frequency range from 0.8 to 47 Hz. It includes both the EEG-dominant and EMG-dominant part of the spectrum."
C. SE (State Entropy) values range from 0 – 100 - false: "Displayed SE values vary between 0 (totally suppressed EEG activity) and 91 (patient awake)"
D. SE is a stable indicator of the effects of hypnotics on the cortex - true by default
E. the RE algorithm filters the EMG (electromyogram) component - false: "The response entropy (RE) is computed over a frequency range from 0.8 to 47 Hz. It includes both the EEG-dominant and EMG-dominant part of the spectrum."
Which capnograph suggests a tension capnothorax during laparoscopic fundoplication?
A. increasing baseline but otherwise normal
B. rebreathing with increasing baseline
C. change in shape of capnograph with decreasing height and became triangular shaped
ie progessively more upsloping phase 3 but falling peak
D. ?
E. slowly rising baseline with normal expiration but slow flattened inspiration
?C
Oxygen cannot be measured by:
A. fuel cell
B. mass spectrometry
C. infra-red spectrophotometry
D. Paramagnetic analysis
E. Raman scattering
C
The output of a diathermy machine does NOT cause patient electrocution because the
A. current is too low
B. current travels on the surface of the body
C. frequency is too high
D. return electrode is never placed between the heart and the operating site
E. voltage is too low
C. frequency is too high - true: "As the frequency of the driving voltage increases, the heating effect increases and the stimulation decreases. Therefore, at frequencies above 100 kHz (i.e. radio frequencies) the effect is entirely heating."
A 60-year-old man with BMI (body mass index) of 30 is having an inguinal hernia repair. He has a
LMA-Proseal® in place and you plan positive pressure ventilation. If the device is properly
seated the maximum pressure in cm of water before leaking during positive pressure ventilation
should be

A. 15
B. 20
C. 30
D. 40
E. 50
C
You are performing a Level 2 check on the anaesthetic machine. The suction bulb at
the common gas outlet does not stay compressed after 10 seconds. The cause of this could be:
A. leak in CO2 absorber
B. loosely seated vaporiser
C. leak in pipeline O2
D. leak in cylinder attachment to anaesthetic machine
E. malfunction in one of the valves of the ventilator circuit
B
In a rotameter the:
A. Bobbin spins inside a tube that has parallel sides
B. Flow is laminar at high flow rates
C. Height of the bobbin is proportional to the pressure drop across the bobbin
D. Pressure drop across the bobbin is constant at varying flows
E. Resistance increases with increasing gas flow
D. Pressure drop across the bobbin is constant at varying flows - true:

"The Rotameter and the Wright peak flow meter are examples of variable orifice flowmeters. Constant pressure in the former is provided by the pressure of the bobbin; the pressure is the ratio of the product of the mass and gravity over the cross sectional area of the bobbin (p = F/A = m a/A). The Rotameter is tapered, and thus the magnitude of the orifice around the bobbin corresponds to the magnitude of gas flow." (Basic science for anaesthetists).
Adult male who is intubated and ventilated, with CVL in situ. Just before surgeon
starts the Line Isolation Monitor alarms about a leak at 5mA. What do you do?
A. stop procedure and move to a safe location
B. sequentially remove non essential monitors from the circuit until fault is identified
C. unplug the CVL to electrically isolate it until fault is identified
D. ensure the patient is earthed
E. Check the diathermy pad
B?
A 50 year old patient is admitted with acute chest pain. An ECG shows tall R waves in lead V1.
The most likely diagnosis is

A. occlusion of the anterior descending coronary artery
B. a lateral myocardial infarction
C. pulmonary infarction
D. posterior myocardial infarction
E. occlusion of the circumflex artery
D. posterior myocardial infarction - true: "Tall lead V1 (tall RV1), defined as an R/S ratio equal to or greater than 1, is not an infrequent occurrence in emergency department patients. This electrocardiographic finding exists as a normal variant in only 1% of patients. Physicians should therefore be familiar with the differential diagnosis for this important QRS configuration. The electrocardiographic entities which can present with this finding include right bundle branch block, left ventricular ectopy, right ventricular hypertrophy, acute right ventricular dilation (acute right heart strain), type a Wolff-Parkinson-White syndrome, posterior myocardial infarction, hypertrophic cardiomyopathy, progressive muscular dystrophy, dextrocardia, misplaced precordial leads, and normal variant." (Mattu et al, Prominent R wave in lead V1: electrocardiographic differential diagnosis. American Journal of Emergency Medicine - Volume 19, Issue 6 2001)
The J-point of the ECG waveform is altered by all of the following EXCEPT

A. hypothermia
B. tachycardia
C. myocardial ischaemia
D. calcium channel blockers
E. digoxin

"The "J-point" is the end of the QRS complex and serves at the take-off point of the ST segment."
D
A diagnosis of pulmonary embolism is most strongly suggested by

A. intraluminal filling defects or vascular cutoffs on angiography
B. PaO2 less than 85 mmHg and an abnormal lung perfusion scan
C. PaO2 less than 85 mmHg and an elevated PaCOz
D. right ventricular hypertrophy with right ventricular strain and right axis deviation
on electrocardiography
E. "unmatched" ventilation-perfusion defects
A
Regarding atrial fibrillation
A. aspirin is as effective as warfarin for the prevention of embolic stroke in elderly patients
B. patients should not receive warfarin therapy unless they have risk factors for embolic stroke
C. patients who are converted to sinus rhythm should be placed on rhythm maintenance therapy
D. rhythm control has NOT been shown to be superior to rate control
E. the best drug for rate control during exercise is digoxin
D
Reverse splitting of the 2nd heart sound is caused by
A. acute pulmonary embolism
B. ASD
C. complete LBBB
D. severe MR
E. pulmonary HT
C. complete LBBB - true: "The most common cause of reversed splitting is complete LBBB, which can be caused either by delayed activation of the LV, as seen in isolated proximal block, or to prolonged mechanical systole (primarily isovolumic contraction time), as seen in proximal or peripheral block invariably associated with significant LV dysfunction"
In the diagnosis of postoperative myocardial infarction

A. echocardiography has little role to play
B. infarct size can be judged from cumulative enzyme release
C. new left bundle branch block on ECG confirms acute infarction
D. serum troponins have high sensitivity and are detectable for up to 5 days
E. ST segment elevation is extremely common
?C
The QT interval may be prolonged by each of the following EXCEPT

A. high intra-thoracic pressure
B. hypothermia
C. magnesium sulphate
D. suxamethonium
E. volatile anaesthetic agents
C (hypomagnesaemia can lead to prolonged QT)
A patient has a history of polyuria and has an elevated serum calcium, normal
parathyroid hormone level and an elevated angiotensin converting enzyme level. The most likely
diagnosis is:

A. milk-alkali syndrome
B. primary hyperparathyroidism
C. malignancy
D. sarcoidosis
E. multiple myeloma
D
Features of anorexia nervosa include each of the following EXCEPT

A. dental caries
B. increased gastric emptying
C. mitral valve prolapse
D. sinus bradycardia
E. superficial parotitis
B - delayed not increased
Addison's disease lab features: Which is NOT correct
A. Ca 2.50 mmol/L
B. BSL 12.0 mmol/L
C. Na 128 mmol/L
D. K 6.1 mmol/L
E. Urea 15mmol/L
A. Ca 2.50 mmol/L - true: "Serum sodium may be low; potassium, calcium, and urea nitrogen may be elevated" (Quick answers)
B. BSL 12.0 mmol/L - false and answer to choose: "Fasting blood glucose may be low" (Quick answers)
C. Na 128 mmol/L - true: "Addison’s disease is characterized by fatigue, weakness, anorexia, nausea and vomiting, cutaneous and mucosal hyperpigmentation, cardiopenia secondary to chronic hypotension, hypovolemia, hyponatremia, and hyperkalemia." Stoelting
D. K 6.1 mmol/L - true: See C.
E. Urea 15mmol/L - true: See A
A 70kg male who has been taking prenisolone 10mg per
day orally for over 12 months, undergoes an uncomplicated
laparotomy and bowel resection. The best management of
his steroid therapy perioperatively, until oral intake resumes, is to administer
A. Hydrocortisone 100mg iv for approximately 24 hours
B. Hydrocortisone 200mg per day for approx 72 hours
C. Hydrocortisone 400mg per day for 72 hrs
D. No steroids
E. 20mg nasogastric prednisolone daily
D or A
A known alcoholic with anorexia and nausea has become jaundiced. His urine is dark and his
faeces pale. He has discomfort in the right hypochondrium. The AST (SGOT) is 2000 IU.l-l,
the alkaline phosphatase 100 IU.l-I and the serum bilirubin is 75 micromol.1-l.
The best treatment would be

A. withdrawal of alcohol
B. expectant
C. operation to remove obstruction (gallstones tumour)
D. urgent liver biopsy
E. endoscopy
A (massively elevated AST consistent with viral hepatitis)
The most important aspect of the peri-operative management of a patient with Gilbert's syndrome is

A. avoidance of fasting
B. avoidance of stress
C. pre-operative transfusion of fresh frozen plasma (FFP)
D. prophylaxis against hepato-renal syndrome
E. recognition of aetiology of the laboratory abnormality
E
In HIV infected patients
A. a CD4 count of 200-400 is typical
B. toxoplasmosis is often a presenting feature
C. cardiomyopathy is a recognised complication
D. regional anaesthesia in obstetrics is contraindicated
E. an epidural blood patch should NOT be used to treat a post-dural puncture headache
C. cardiomyopathy is a recognised complication - true: "Myocarditis, progressing to dilated cardiomyopathy, is common and may be caused by infection with Cryptococcus, coxsackie B, virus, CMV, Aspergillus species as well as lymphoma and HIV itself." (Anaesthesia and critical care for patients with HIV infection, Continuing Education in Anaesthesia, Critical Care & Pain | Volume 5 Number 5 2005)
von Willebrands type 1:
A. Variable increase in bleeding time
B. Increased aPTT
C. Increased PT
D. Haemarthroses
E. Delayed bleeding after trauma
A
Findings in a patient with serotonin syndrome include each of the following EXCEPT

A. clonus
B. diaphoresis
C. hyperreflexia
D. miosis
E. tachycardia
D

Neuromuscular hyperactivity; tremor, clonus, myoclonus, hyper-reflexia and (in the advanced stage) pyramidal rigidity.

Autonomic hyperactivity; diaphoresis, fever, tachycardia and tachypnoea.

Altered mental status; agitation, excitement and (in the advanced stage) confusion.
Carcinoid syndrome may present as:

A. SVT
B. Mitral regurgitation
C. Hypoglycaemia
D. Renal failure
E. Splenomegaly
supraventricular tachycardia - true: "5-HT has little, if any direct effect on the heart. With elevated levels, however, positive chronotropic and inotropic myocardial effects may occur, mediated by the release of noradrenaline" (Miller 6th ed p.1109)
In chronic obstructive pulmonary disease (COPD), the variable most closely associated with prognosis is
A. arterial carbon dioxide partial pressure (PaC02)
B. arterial oxygen partial pressure (Pa02)
C. forced expiratory volume in one second (FEV1)
D. forced vital capacity (FVC)
E. response to bronchodilators
C

'FEV1 is the parameter traditionally used for prognosis' UTD

'30% of patients (with FEV <35% of predicted) are dead within 1 year and 95% within 10 years ' Merck
Unequal consolidation on a CXR can be caused by all except

A. pulmonary oedema
B. pneumonia
C. pulmonary haemorrhage
D. pulmonary infarction
E. pulmonary effusion
Unequal consolidation on a CXR can be caused by all except

A. pulmonary oedema
B. pneumonia
C. pulmonary haemorrhage
D. pulmonary infarction
E. pulmonary effusion
An INCORRECT statement regarding the management of hypocalcaemia is that

A. Correcting a respiratory or metabolic alkalosis increases the level of ionised calcium
B. Calcium should be administered via a central vein as it is irritant to peripheral veins
C. Acidosis will decrease calcium binding to albumin and therefore increase ionised calcium
D. When calcium alone is NOT sufficient for control of hypocalcaemia, Vitamin D metabolites
can be added
E. Calcium chloride has been shown to be superior to calcium gluconate
A. Correcting a respiratory or metabolic alkalosis increases the level of ionised calcium - true: "Initial therapy (of hypocalcaemia in adults consists of correction of any coexisting respiratory or metabolic alkalosis" (Aguilera and Vaughan, "Calcium and the anaesthetist", Anaesthesia 2000, 55:779-790)
B. Calcium should be administered via a central vein as it is irritant to peripheral veins - true: "Calcium is irritating to veins and should be diluted before administration and all solutions are best administered through a central vein." (Aguilera and Vaughan, "Calcium and the anaesthetist", Anaesthesia 2000, 55:779-790)
C. Acidosis will decrease calcium binding to albumin and therefore increase ionised calcium - true: "Acidosis decreases calcium binding to albumin thus increasing ionised calcium, while alkalosis increases binding (calcium shifts onto the albumin molecule), producing a subsequent reduction in the ionised calcium." (Aguilera and Vaughan, "Calcium and the anaesthetist", Anaesthesia 2000, 55:779-790)
D. When calcium alone is NOT sufficient for control of hypocalcaemia, Vitamin D metabolites can be added - true: "When calcium alone is not sufficient for control of hypocalcaemia, vitamin D metabolites can be added." (Aguilera and Vaughan, "Calcium and the anaesthetist", Anaesthesia 2000, 55:779-790)
E. Calcium chloride has been shown to be superior to calcium gluconate - false and answer to choose: "When equivalent calcium doses are administered, both preparations are equally efficacious in restoring the calcium level to normal ( Table 54-12 ). In this regard, calcium gluconate is especially advantageous in peripheral venous administration because extravasated calcium chloride solution can result in severe tissue destruction." (Miller Ch 54)
Patho-physiological features of patients with morbid obesity include

A. a blood volume:body weight ratio which is similar to that of patients with normal body weight
B. an increased blood pressure and systemic vascular resistance compared to that of patients with
normal body weight
C. decreased gastric motility due to increased gastrin secretion
D. cardiac pathology resulting from excess body mass and increased metabolic demand
E. cardiac pathology resulting mainly from fatty infiltration or fatty change of the heart
A. a blood volume:body weight ratio which is similar to that of patients with normal body weight - false: "Total blood volume is increased in the obese but on a volume/weight basis is less than that in non-obese individuals" (Adams and Murphy, Obesity in anaesthesia and intensive care, BJA 2000; 85:91-108)
B. an increased blood pressure and systemic vascular resistance compared to that of patients with normal body weight - false: "hypertension is presumably caused by an increased cardiac output forced into an unaltered peripheral resistance" (Oberg and Poulsen, Obesity: an anaesthetic challenge, Acta Anaesthesiologica Scandinavica 1996, 40: 191-200)
C. decreased gastric motility due to increased gastrin secretion - false: gastrin is a prokinetic hormone
D. cardiac pathology resulting from excess body mass and increased metabolic demand - true by default
E. cardiac pathology resulting mainly from fatty infiltration or fatty change of the heart - false: "The morbidly obese individual is at risk of a specific form of obesity-induced cardiac dysfunction, although the belief is that this is secondary to fatty infiltration of the heart ('cor adiposum') is no longer valid." (Adams and Murphy)
In patients with chronic renal failure there is

A. increased calcium absorption
B. increased phosphate excretion
C. increased vitamin D3 production
D. increased osteoclastic activity
E. decreased osteoblastic activity
A. increased Ca absorption - false: "Renal osteodystrophy is a complication of chronic renal failure, reflecting the complex interaction of secondary hyperparathyroidism and decreased vitamin D production by the kidneys. As the GFR decreases, there is a parallel decrease in phosphate clearance and an increase in the serum phosphate concentrations that result in reciprocal decreases in serum calcium concentrations. Hypocalcaemia stimulates PTH secretion, which leads to bone resorption and calcium release. As a result of decreased renal production of Vitamin D by the kidneys, intestinal absorption of calcium is impaired, which also leads to hypocalcaemia, stimulation of PTH release, and bone resorption." (Stoelting's Anesthesia and Coexisting Disease, 5th ed, p.332)
B. increased PO4 absorption - false: see above
C. increased vit D3 production - false: see above
D. increased osteoclastic activity - true
E. decreased osteoblastic activity - false
Consider the following blood gases. Normal ranges are in brackets.

pH 7.28
PaCO2 36
Bicarbonate 18 mmol.l-1 (18-25)
Base excess -7 mmol.l-1 (-4- +3)
Na+ 142 mmol.l-1 (135-145)
Cl- 112 mmol.l-1 (98-110)

These blood gases are consistent with

A. acute renal failure
B. diabetic ketoacidosis
C. ethylene glycol overdose
D. intraoperative infusion of 6 litres of normal saline
E. salicylate overdose
D
A 40y.o. man with Marfan's syndrome is to undergo thoraco-abdominal aortic
reconstruction for chronic aortic dissection. An intrathecal catheter is
inserted. The purpose of this is to
A. allow drainage of CSF
B. allow intrathecal admin of metabolic substrates
C. allow intrathecal admin of neuroprotective drugs
D. cool the spinal cord
E. facilitate spinal cord function monitoring
A. allow drainage of CSF - true
B. allow intrathecal admin of metabolic substrates
C. allow intrathecal admin of neuroprotective drugs
D. cool the spinal cord
E. facilitate spinal cord function monitoring
For a patient positioned for left lateral for posterolateral incision for thoracotomy
A. right brachial plexus is at risk if shoulder flexed > 90 degrees
B. radial nerve is most common injury
C. flex the non-dependant leg, and straighten the dependant leg
D. place axillary roll under chest to improve surgical exposure
E. the neck should be extended to allow access to the airway
Answer: A? "Compression also plays a predominant role in injury with the patient in the lateral decubitus position when the plexus is compressed against the thorax by the humeral head" From R. J. Sawyer, M. N. Richmond, J. D. Hickey, J. A. Jarrratt (2000)Peripheral nerve injuries associated with anaesthesia, Anaesthesia 55 (10), 980–991.

B is false "Frequently damaged nerves include the branches of the brachial plexus and the ulnar and common peroneal nerves. The radial nerve is damaged infrequently" From B. E. Tuncali, B. Tuncali, B. Kuvaki, O. Cinar, A. Doğan, Z. Elar (2005) Radial nerve injury after general anaesthesia in the lateral decubitus position* Anaesthesia 60 (6), 602–604.

C. I think it is the other way round, ie. flex the dependent leg, and straighten the non dependent leg? But i can't be 100% sure now. Maybe it is true (for recovery position yes, but in surgery....)

D. "An axillary roll is placed to minimize the risk of brachial plexus injury" [1]--Life is Beautiful 07:26, 9 Jul 2007 (EDT)

Re B. Wouldn't the long thoracic nerve (by scalpel) be the most commonly injured, or does this not count?Tommytank 07:09, 20 Jul 2007 (EDT)

C: definitely wrong, should be opposite (I've checked)
Relative contraindications to mediastinoscopy include
A. Cervical spondylosis
B. Emphysema
C. Mediastinal lymphadenopathy
D. Poor left ventricular function
E. Superior vena cava syndrome
?E
In relation to obstetric haemorrhage

A. amniotic fluid embolism is unlikely to present as unexplained haemorrhage
B. coagulopathy is uncommon, when severe abruption leads to maternal shock and fetal death
C. the risk of placenta accreta, but NOT placenta previa, increases with an increasing
number of caesarean sections
D. treatment of uterine atony with prostaglandins is rarely assosciated with maternal
adverse effects
E. intravenous magnesium may facilitate replacement of an inverted uterus
A. amniotic fluid embolism is unlikely to present as unexplained haemorrhage
B. coagulopathy is uncommon, when severe abruption leads to maternal shock and fetal death
C. the risk of placenta accreta, but NOT placenta previa, increases with an increasing number of caesarean sections
D. treatment of uterine atony with prostaglandins is rarely assosciated with maternal adverse effects
E. intravenous magnesium may facilitate replacement of an inverted uterus - true: "Uterine relaxation may be necessary to replace the uterus; β-sympathomimetic agents, magnesium, and nitroglycerin all have been used to achieve this goal." (Miller 7th ed Ch 69)
Supine hypotension during late pregnancy is associated with

A. a rise in the systemic vascular resistance
B. a rise in the cardiopulmonary blood volume
C. increased heart rate
D. stable stroke volume
E. a rise in the cardiac index
C or A
Drugs that may be used for the management of heart failure, secondary to dilated cardiomyopathy
in pregnancy, include each of the following EXCEPT

A. ACE (angiotensin-converting enzyme) inhibitor
B. beta-blockers
C. digoxin
D. loop diuretics
E. nitrates
A class D - ACE inhibitors have also been associated with fetal death in utero.

There is a potential risk of fetal hypotension, decreased birthweight and decreased renal perfusion or anuria in the fetus from in utero exposure to ACE inhibitors. Oligohydramnios in the mother has also been reported, presumably representing decreased renal function in the fetus
A pregnant woman is 36 weeks gestation and complains of progressive dyspnoea.
The sign which would prompt you to further investigate would be

A. peripheral oedema
B. increased JVP
C. S3
D. ejection systolic murmur
E. orthopnea
?B
Best evidence in obstetric anaesthesia supports each of the following
assertions EXCEPT

A. colloid prevents hypotension from regional anaesthesia more effectively
than crystalloid
B. fentanyl added to spinal bupivacaine for caesarean section has no
influence on the incidence of intraoperative nausea
C. high doses of ephedrine (>15 mg) are more likely to cause hypertension
than prevent hypotension
D. in labour, combined spinal-epidural analgesia is associated with faster
onset and greater maternal satisfaction than epidural analgesia
E. in nulliparous women, epidural analgesia in labour, compared with
intravenous opioid analgesia, does not increase caesarean section rate
?B
Analgesic requirements during labour are reduced by each of the following except
A. Acupressure
B. Acupuncture
C. Hypnosis
D. One to one support by midwife
E. TENS
A. Acupressure - false: "Published in the Cochrane Library collection of evidence-based literature was a systematic review of complementary and alternative therapies for pain management in labor. This meta-analysis of 14 studies evaluated the efficacy of acupuncture, acupressure, audioanalgesia, aromatherapy, hypnosis, massage, and relaxation for labor analgesia. Only two therapies were found to be beneficial, acupuncture and hypnosis." (Macarthur, Gerard W. Ostheimer “What’s New in Obstetric Anesthesia” Lecture, in Anesthesiology 2008; 108:777–85)
B. Acupuncture - true: "Acupuncture decreases the need for analgesics" (ANZCA and FPM, Acute Pain Management: Scientific Evidence, 2E, 2005, p229)
C. Hypnosis - true: "Hypnosis used in labour also leads to a decreased requirement for pharmacological analgesia" (ANZCA and FPM, Acute Pain Management: Scientific Evidence, 2E, 2005, p229)
D. One to one support by midwife - true: "Continuous or one-to-one support by a midwife or trained layperson during labour reduces analgesic use" (ANZCA and FPM, Acute Pain Management: Scientific Evidence, 2E, 2005, p229)
E. TENS - true but possibly dodgy evidence: "Randomised controlled trials provide no compelling evidence for TENS having any analgesic effect during labour. Weak positive effects in secondary (analgesic sparing) and tertiary (choosing TENS for future labours) outcomes may be due to inadequate blinding causing overestimation of treatment effects" (Carroll et al, Transcutaneous electrical nerve stimulation in labour pain: a systematic review, BJOG February 1997, Vol. 104, pp. 169-175)
The most effective method of deep venous thrombosis (DVT) prophylaxis for a fifty-year-old
woman presenting for anterior resection for cancer of the colon would be

A. electrical calf stimulation
B. Dextran 70 infusion
C. graduated compression stockings
D. intermittent pneumatic leg compression
E. low dose heparin (5000 units bd)
A. electrical calf stimulation - false: Mechanical forms of VTE prevention are "considered generally less efficient than pharmacologic intervention." (Nielsen and Asmis, Hypercoagulability in the perioperative period, in Best Practice & Research Clinical Anaesthesiology Volume 24, Issue 1, March 2010, Pages 133-144)
B. Dextran 70 infusion
C. graduated compression stockings - false: See A
D. intermittent pneumatic leg compression - false: See A
E. low dose heparin (5000 units bd) - best answer
In regards to laparotomy and hemicolectomy, the least effective way to minimize post op surgical infection is:
A. Aggressive peri-operative fluid management
B. Timely administration of prophylactic antibiotics
C. Perioperative hyperoxia
D. Avoidance of hypothermia
E. Avoidance of blood transfusion
A. Aggressive peri-operative fluid management - false and answer to choose: "At this time, it seems that aggressive fluid resuscitation does not decrease the incidence of SSI, and maintenance of euvolemia is recommended based on clinical parameters." (The Anesthesiologist's Role in the Prevention of Surgical Site Infections in Anesthesiology Issue: Volume 105(2), August 2006, pp 413-421)
B. Timely administration of prophylactic antibiotics - true: "Perhaps the simplest and most effective role the anesthesiologist can play in the prevention of SSIs is ensuring the administration of appropriate antimicrobial prophylaxis" (The Anesthesiologist's Role in the Prevention of Surgical Site Infections in Anesthesiology Issue: Volume 105(2), August 2006, pp 413-421)
C. Perioperative hyperoxia - jury out: "These facts have led some researchers to hypothesize that providing supplemental oxygen during the perioperative period would lead to higher oxygen tensions in the wound and a decrease in the incidence of SSIs." (The Anesthesiologist's Role in the Prevention of Surgical Site Infections in Anesthesiology Issue: Volume 105(2), August 2006, pp 413-421)
D. Avoidance of hypothermia - true: "Although a few studies have not shown an increased risk of infection in hypothermic patients,8 most studies, including those reviewed here, implicate mild intraoperative hypothermia as a risk factor for postoperative SSI." (The Anesthesiologist's Role in the Prevention of Surgical Site Infections in Anesthesiology Issue: Volume 105(2), August 2006, pp 413-421)
E. Avoidance of blood transfusion - true: "many recent prospective studies have linked perioperative transfusion to infection in multiple surgical populations" (The Anesthesiologist's Role in the Prevention of Surgical Site Infections in Anesthesiology Issue: Volume 105(2), August 2006, pp 413-421)
Post liver resection, the time when prothrombin time derangement is maximal is:
A. day 1-2
B. day 3-4
C. day 4-5
D. day 6-7
E. unpredictable time after surgery
A
Phosphate-containing bowel preparations for colonoscopy are contra-indicated in frail, elderly
patients because of the

A. frequent nausea, bloating and abdominal pain
B. risk of hypercalcaemia
C. risk of hypokalaemia
D. risk of hypomagnesaemia
E. risk of large fluid shifts
E (large fluid shifts) is the primary reason. "Phosphate preparations have the potential to cause electrolyte disturbances including serious hyperphosphataemia and hypocalcaemia; deaths have been reported. Sodium phosphate must therefore be avoided in patients with impaired renal function and used with great care in the presence of congestive cardiac failure because of the potential large fluid shifts. The frail, elderly and the very young are particularly at risk of fluid and electrolyte complications and alternative preparations should be used." Australian Prescriber
Blunt liver trauma can be treated non surgically if
A. No peritoneal signs
B. Low Grade injury on CT scan
C. Severe COPD
D. Haemodynamically stable
E. US confirms <500mls peritoneal fluid collection
D
The intraoperative hypothermia for aneurysm surgery trial (IHAST) showed that cooling to a target
temperature of 33°C

A. did NOT improve neurological outcome in WFNS (World Federation of Neurosurgical Surgeons)
grade I-III patients
B. did NOT improve neurological outcome in WFNS grade IV-V patients
C. improved neurological outcome in WFNS grade I-III
D. improved neurological outcome in WFNS grade III
E. improved neurological outcome in WFNS grade IV-V
A


BACKGROUND: Surgery for intracranial aneurysm often results in postoperative neurologic deficits. We conducted a randomized trial at 30 centers to determine whether intraoperative cooling during open craniotomy would improve the outcome among patients with acute aneurysmal subarachnoid hemorrhage.

METHODS: A total of 1001 patients with a preoperative World Federation of Neurological Surgeons score of I, II, or III ("good-grade patients"), who had had a subarachnoid hemorrhage no more than 14 days before planned surgical aneurysm clipping, were randomly assigned to intraoperative hypothermia (target temperature, 33 degrees C, with the use of surface cooling techniques) or normothermia (target temperature, 36.5 degrees C). Patients were followed closely postoperatively and examined approximately 90 days after surgery, at which time a Glasgow Outcome Score was assigned.

RESULTS: There were no significant differences between the group assigned to intraoperative hypothermia and the group assigned to normothermia in the duration of stay in the intensive care unit, the total length of hospitalization, the rates of death at follow-up (6 percent in both groups), or the destination at discharge (home or another hospital, among surviving patients). At the final follow-up, 329 of 499 patients in the hypothermia group had a Glasgow Outcome Score of 1 (good outcome), as compared with 314 of 501 patients in the normothermia group (66 percent vs. 63 percent; odds ratio, 1.14; 95 percent confidence interval, 0.88 to 1.48; P=0.32). Postoperative bacteremia was more common in the hypothermia group than in the normothermia group (5 percent vs. 3 percent, P=0.05).

CONCLUSIONS: Intraoperative hypothermia did not improve the neurologic outcome after craniotomy among good-grade patients with aneurysmal subarachnoid hemorrhage.
Each of the following statements regarding vasospasm occurring after SAH is true EXCEPT:
A. cerebral vessel narrowing, demonstrated angiographically, can be reversed 30-40% of the time by administering nimodipine
B. combination therapy resulting in HT, hypervolaemia and haemodilution is a mainstay for prevention and treatment
C. other cause of neurological deterioration (such as hydrocephalus) need to be excluded before making the diagnosis
D. sequential TC doppler measurement may detect those patients at risk
E. the peak incidence is 7-10 days after the SAH
?A
Post grade II SAH in 50 year old woman who has just returned from successful endoluminal
coiling. The best thing to include in subsequent management to prevent vasospasm would be:
A. at least 3 L of IV normal saline per day
B. IV or oral magnesium supplementation
C. maintain systolic blood pressure above 160mmHg
D. Keep intubated and ventilate for 24 hours before de-sedating
E. cool to 33 degrees
?A ?C
A patient is having posterior fossa surgery in the sitting position.
Which of the following changes would make you suspect a venous air embolism:
A. ↓PAP, ↓ETCO2, ↑PaCO2, ↓CVP
B. ↓PAP, ↑ETCO2, ↓PaCO2, ↓CVP
C. ↑PAP, ↓ETCO2, ↑PaCO2, ↑CVP
D. ↑PAP, ↑ETCO2, ↓PaCO2, ↑CVP
E. ↑PAP, ↓ETCO2, ↓PaCO2, ↑CVP
C. ↑PAP, ↓ETCO2, ↑PaCO2, ↑CVP - true "End tidal carbon dioxide falls... In 25% of patients the CVP is elevated and the pulmonary artery pressure rises in 50%. Arterial blood gases may reveal hypoxaemia and, less commonly hypercarbia." (S Webber , J Andrzejowski , and G Francis Gas embolism in anaesthesia BJA CEPD Reviews 2: 53-57.)
A previously healthy 20-year-old male is undergoing open reduction and internal fixation of a
fractured tibia. The limb was exsanguinated and the tourniquet appropriately applied and inflated
to 250 mmHg prior to surgery. A small amount of continued bleeding is noted after surgical
exposure. The tourniquet still seems appropriately inflated. The patient's blood pressure
is 110/70 mmHg. You should

A. accept that a tourniquets does not stop all bleeding
B. check the patient's coagulation profile
C. inflate the tourniquet to a higher pressure
D. reduce the patient's blood pressure
E. re-exsanguinate the limb and re-apply the tourniquet
E. re-exsanguinate the limb and re-apply the tourniquet - probably the best answer: "Common causes of intraoperative bleeding include incomplete exsanguination of the limb and a poorly fitting or under-pressurized cuff. Intraoperative bleeding may also be caused by blood entering through the intramedullary vessels of long bones." (Arterial Tourniquets, Continuing Education in Anaesthesia, Critical Care & Pain 2009 9(2):56-60)
A well 65 year old is having a total hip replacement under general anaesthesia with sevo/N2O/fentanyl. BP is 130/70.
Before the surgeon commences reaming and cementing, the best thing to do is:
A. Induce hypotension
B. Raise BP with vasopressors
C. Turn off nitrous
D. Give steroids
E. Give heparin 5000u
C. Turn off nitrous - maybe and potentially the best answer: "Whether early administration of oxygen actually prevents the onset of the syndrome by preventing hypoxaemia, further catecholamine response and fat mobilisation remains unclear." (Mellor and Soni, Fat Embolism Review Article, Anaesthesia, 2001, 56, pages 145-154) Assuming that turning off the nitrous means increasing the FiO2, this is a relatively simple procedure to do with very little downside.
The absorption of fluid into the circulation during transurethral prostatectomy (TURP)
is NOT related to

A. prostate size
B. height of the irrigation fluid bag
C. duration of surgery
D. surgical technique
E. type of irrigation fluid
E. type of irrigation fluid - false and the answer to choose: As described below, fluid is absorbed through the venous networks rather than across a semi-permeable membrane. The makeup of the fluid DOES affect the likelihood of TURP syndrome as it may alter electrolyte homeostasis, BUT it will not affect the volume of fluid absorbed.
In the recovery room, following general anaesthesia for renal transplant surgery, your patient is
found to have a serum potassium concentration of 6 mmol.l-1, despite having a normal potassium
concentration pre-operatively. His oxygen saturation is 96% on approximately 40% oxygen via a
Hudson mask. He is still unconscious, but breathing spontaneously at 8 breaths per minute. The most
likely cause of his hyperkalaemia is

A. beta-blockers which he received peri-operatively
B. catabolic stress of surgery
C. opioid induced narcosis causing carbon dioxide retention
D. renal graft failure
E. washed red blood cell transfusion, which he received intraoperatively
A. beta-blockers which he received peri-operatively - false
B. catabolic stress of surgery - unlikely
C. opioid induced narcosis causing carbon dioxide retention - most likely
D. renal graft failure - unlikely as the graft probably hasn't even taken yet
E. washed red blood cell transfusion, which he received intraoperatively - maybe, but washed red cells have a lower potassium content
The physiological change most likely to be associated with infrarenal cross-clamping of the aorta is

A. decreased coronary blood flow
B. decreased renal blood flow
C. increased cardiac output
D. increased heart rate
E. reduction in myocardial wall motion abnormalities
B. decreased renal blood flow - true: "Although it is clear that a suprarenal cross-clamp will decrease renal blood flow up to 80% an infrarenal cross-clamp also causes a decrease in renal cortical blood flow with an associated decrease in GFR." (Yao and Artusio, 6E Ch 11 p 291)
In a patient with an intra-orbital haemorrhage, following local anaesthetic injection, the adequacy
of ocular perfusion is best assessed by

A. angiography
B. direct ophthalmoscopy
C. indirect ophthalmoscopy
D. intra-ocular pressure tonometry
E. palpation of the globe by an experienced clinician
???B
A six year old boy requires return to theatre for arrest of post-tonsillectomy haemorrhage.
When anaesthetising for this procedure it is important to:
A. avoid sedative premedication prior to induction
B. avoid volatile anaesthesic agents
C. have duplicate suction apparatus and ETTs available
D. use an uncuffed endotracheal tube in this age group
E. RSI
From the recent CEACP review, regarding the bleeding tonsil: "Before induction, in addition to the standard equipment, a selection of laryngoscope blades, smaller than expected tracheal tubes, and two suction catheters should be immediately available". Looks like C
A 60y.o. man with a 45 pack year history of smoking and heavy drinking
presents with stridor. He has had a hoarse voice for about 4 months. He
has been waking at night for a week with difficulty breathing relieved
by sitting up. He is sitting upright in bed with moderate inspiratory
stridor. His SaO2 on air is 95%. The next step in his management should be
A. awake fibreoptic intubation
B. CT scan of the neck
C. examination under anaesthesia following gaseous induction
D. nasenoscopy under topical anaesthesia
E. tracheostomy under LA
D
Block of the maxillary nerve in the pterygopalatine fossa causes ipsilateral
analgesia of:
A. Upper molar teeth
B. Upper incisor teeth
C. Hard palate
D. The posterior part of the lateral wall of the nose
E. Nasal septum
A. Hard palate - true: "The hard and soft palate is innervated by the palatine branches; the greater (anterior) and lesser (middle and posterior) palatine nerves. After descending through the pterygopalatine canal, the greater palatine nerve exits the greater palatine foramen onto the hard palate. The nerve provides sensory innervation to the palatal mucosa and bone of the hard and soft palate. The lesser palatine nerves emerge from the lesser palatine foramen to innervate the soft palate and tonsillar region." (Nysora.com[1])
B. Nasal septum - true: The nasopalatine nerve is a branch of the maxillary nerve. "The nasopalatine nerve courses medially through the sphenopalatine foramen and then over the roof of the nose to reach the nasal septum. Here, it turns forward and downward and travels along the septum to reach the incisive canal, emerging behind the upper incisors. It carries sensation from the nasal septum and the anterior part of the hard palate in an area just behind the upper incisors." (Current Dx and Tx in Otolaryngology)
C. The posterior part of the lateral wall of the nose
D. Upper incisor teeth - true: "The anterior superior alveolar nerve arises at the anterior end of the infraorbital canal and descends in the anterior maxillary wall to innervate the upper canine and incisor teeth." Note that this is distal to the pterygopalatine fossa
E. Upper molar teeth - also true: "The posterior superior alveolar nerve, which may be double, descends over the posterior surface of the maxilla, then enters the posterior dental canal (which again may be double) on the posterior aspect of the maxilla, and gives branches to each molar tooth. In addition, branches are given off to the mucosa of the maxillary sinus."
Advantages of local anaesthesia for middle ear surgery compared with general anaesthesia
do NOT include

A. avoidance of problems with middle ear pressure
B. a drier surgical field
C. ability to monitor facial nerve integrity
D. ability to monitor hearing
E. substantial reduction in post-operative nausea and vomiting
C. ability to monitor facial nerve integrity - false and answer to choose: CN VII can be monitored under GA or LA, but LA frequently tracks around to block the facial nerve rendering monitoring impossible
In performing a retrobulbar block, the complication of brainstem anaesthesia:
A. Is not associated with contralateral amaurosis
B. Has an incidence of approximately 1 in 2000 blocks
C. Becomes clinically apparent within 2 to 15 minutes
D. Usually takes 4 to 6 hours to resolve
E. Is associated with increased auditory acuity
C

incidence with retrobulbar is 0.3-0.8%, is assoc with CL amaurosis, apparent within 10 mins, lasts 2-3 hours, decreased auditory acuity
A patient requires forefoot amputation. You wish to block the most peripheral nerves
to give complete anaesthesia for the procedure. Best nerves to be blocked include:
A. Medial and lateral plantar
B. common peroneal and tibial
C. sciatic and saphenous
D. sural and sciatic
E. deep peroneal and superficial peroneal
From NYSORA website "An ankle block is essentially a block of four branches of the sciatic nerve (deep and superficial peroneal, tibial and sural nerves) and one cutaneous branch of the femoral nerve (saphe-nous nerve). An ankle block is a basic, peripheral nerve block technique. It is simple to perform, essentially devoid of systemic complications, and highly effective for a wide variety of proce-dures on the foot and toes. As such, this technique should be in the armamentarium of every anesthesiologist. In our institution, an ankle block is most commonly used in podiatry surgery and foot and toes debridement or amputation."
The nerve which supplies the lobule of the ear is the

A. greater auricular
B. lesser occipital
C. auricular branch of the vagus
D. auriculo-temporal
E. greater occipital
A. great auricular - true: "The great auricular nerve (C2,3) is the largest cutaneous branch of the cervical plexus. It hooks around the mid-point of the posterior border of sternocleidomastoid, then passes across it in the direction of the angle of the mandible. On this muscle it breaks up into three terminal branches.
1 Auricular - supplying the lower two-thirds of the medial aspect of the external ear and the lateral surface of the lobule.
2. Mastoid - to the skin over the mastoid process.
3 Facial - to the skin over the masseter and the parotid gland." (Anatomy for anaesthetists p.148)
B. lesser occipital - false: "The lesser occipital nerve (C2) hooks around the spinal accessory nerve (XI), then ascends along the posterior border of the sternocleidomastoid. It pierces the deep fascia in the upper part of the posterior triangle, then splits up into three branches.
1 Auricular - to the upper third of the medial aspect of the external ear.
2 Mastoid - to the skin over the mastoid process.
3 Occipital - to the occipital area immediately above and behind the mastoid."
C. auricular branch of vagus - false: Of the vagus nerve, "the auricular branch also originates from the superior ganglion, then enters a tiny canal on the lateral wall of the jugular fossa that leads it into the temporal bone, from which the nerve emerges again between the mastoid process and the tympanic plate. It supplies the medial aspect of the auricle, the external auditory meatus and the outer surface of the tympanic membrane. It communicates with the facial nerve both in the petrous temporal bone and again with the posterior auricular branch of VII on emerging from the bone."
D. auriculo-temporal - false: A branch of the mandibular division of the trigeminal nerve. "The auriculotemporal nerve gives off the following branches:
1 Auricularato the skin of the tragus and the adjacent lateral aspect of the helix.
2 Superficial temporalathis ramifies over the skin of the temporal region and the lateral aspect of the scalp.
3 Branches to the external auditory meatusa(usually two), which supply the skin of the meatus and the tympanic membrane.
4 Articularato the temporomandibular joint.
5 Parotidawhich conveys secretomotor, sympathetic and sensory fibres to the salivary gland (see otic ganglion, p. 265)."
E. greater occipital - false: The medial branch of the dorsal ramus of C2. "The medial branch is the greater occipital nerve, which pierces semispinalis capitis and then trapezius, is joined by a filament from the medial branch of C3 and then ascends medial to the occipital artery to supply the skin of the occipital region as far as the vertex. Anteriorly, it overlaps with the lesser occipital nerve, derived from the anterior primary ramus of C2. It gives a branch to semispinalis (Fig. 110)."
The largest and most direct branch of the internal carotid artery is the

A. ophthalmic
B. anterior cerebral
C. middle cerebral
D. posterior cerebral
E. choroidal
C

MCA
The carotid sinus derives its nerve supply from the

A. vagus nerve
B. glossopharyngeal nerve
C. ansa cervicalis (hypoglossi)
D. middle cervical ganglion
E. stellate ganglion
B. glossopharyngeal nerve - true: "The glossopharyngeal nerve contains sensory fibres for the pharynx, the tonsillar region and the posterior one-third of the tongue (including the taste buds), motor fibres for the stylopharyngeus muscle and secretomotor fibres for the parotid gland. It also innervates the carotid sinus and body... The carotid sinus is a bulge at the start of the internal carotid artery. Here the arterial wall is thin and has a particularly rich nerve supply from the glossopharyngeal nerve."
The innervation of the human larynx is such that

A. the internal laryngeal branch of the superior laryngeal branch of the vagus supplies
the lingual surface of the epiglottis
B. in the cadaveric position the cords are fully abducted
C. the recurrent laryngeal nerve supplies all the intrinsic muscles of the larynx
D. the glossopharyngeal nerves are sensory to the laryngeal mucous membrane above the level
of the vocal cords
E. cord paralysis can be produced by a distended endotracheal cuff in the larynx compressing
a branch of the recurrent laryngeal nerve against the thyroid cartilage
E. cord paralysis can be produced by a distended endotracheal cuff in the larynx compressing a branch of the recurrent laryngeal nerve against the thyroid cartilage - true: "Compression of the anterior branch of the recurrent laryngeal nerve between the cuff of the endotracheal tube and the posterior part of the thyroid cartilage was the likely mechanism" (of paralysis) [Anaesth Intensive Care. 2004 Jun;32(3):417-8. Bilateral adductor vocal cord paresis following endotracheal intubation for general anaesthesia.]
Stellate ganglion is where:
A. at the level of the body of C6 (spine of C6)
B. posterior to the brachial plexus sheath
C. anterior to the dome of the pleura
D. anterior to the thoracic duct
E. anterior to scalenius anterior
?E
The best predictor of poor outcome for a peri-operative ulnar nerve injury is

A. a delay in symptom onset to more than 48 hours postoperatively
B. association with anaesthesia lasting more than 2 hours
C. association with a brachial plexus block
D. presence of bilateral injury
E. presence of mixed sensory and motor deficit
E. presence of mixed sensory and motor deficit - true: "Patients with only sensory deficits 3 months after their procedures had a greater chance of complete recovery at 1 yr than did patients with mixed sensory and motor deficits." (Warner et al, "Ulnar Neuropathy" in Anesthesiology, 1994, 81:1332-1340)
The median nerve

A. can be blocked at the elbow immediately lateral to the brachial artery
B. can be blocked at the wrist between palmaris longus and flexor carpi ulnaris
C. can be blocked at the wrist medial to flexor carpi ulnaris
D. is formed from the lateral, medial, and posterior cords of the brachial plexus
E. provides sensation to the radial half of the palm
A. can be blocked at the elbow immediately lateral to the brachial artery - false: lies medial
B. can be blocked at the wrist between palmaris longus and flexor carpi ulnaris - false: lies between FCR and PL
C. can be blocked at the wrist medial to flexor carpi ulnaris - false
D. is formed from the lateral, medial, and posterior cords of the brachial plexus - false
E. provides sensation to the radial half of the palm - true
What do C6/7 motor function do
A. flex/extension of fingers
B. flex /extend wrist
C. shoulder ext rotation / abduction
D. elbow pronation/supination
E. flexion at elbow
A c7,8
B true Last p 16
C c5
D Pronation = c7,8 Supination = c5
E c5,c6
At the wrist joint
A. the median nerve lies between the tendons of palmaris longus and flexor carpi radialis
B. the median nerve lies medial to the tendon of flexor carpi radialis
C. the radial artery is usually palpable just medial to the tendon of flexor carpi radialis
D. the ulnar artery and nerve enter the hand by passing deep to the flexor retinaculum
A. the median nerve lies between the tendons of palmaris longus and flexor carpi radialis - true: The median nerve lies superficially "with the tendon of FCR lying laterally and those of FDS and palmaris longus medially" (Anatomy for Anaesthetists 7ED p. 180)
B. the median n. lies medial to the tendon of FCU - false: See fig 124; FCU lies medial to the median nerve
C. the radial a. is usually palpable just medial to the tendon of FCU: false: radial artery is on the lateral side of the wrist
D. the ulnar artery and nerve enter the hand by passing deep to the flexor retinaculum - partly true: "The median nerve passes into the hand through the carpal tunnel" (Anatomy for anaesthetists) but the ulnar artery passes superficially.
Pre-ganglionic sympathetic fibres pass to the

A. otic ganglion
B. carotid body
C. ciliary ganglion
D. coeliac ganglion
E. all of the above
D
The peak effect of oral midazolam as a premedication in children occurs after

A. 10 - 15 minutes
B. 20 - 30 minutes
C. 35 - 45 minutes
D. 50 - 60 minutes
E. 65 - 75 minutes
B. 20 - 30 minutes - true: "The dose of oral midazolam... usually results in a satisfactorily sedated child in approximately 10-15 min with a peak effect occurring at approximately 20-30 min, with minimal to no delay in recovery, even for brief procedures." (Cote, Preoperative preparation and premedication, BJA 1999, 83:16-28)
A six-week-old baby is booked for elective right inguinal hernia repair.
An appropriate fasting time is
A. 2 hours for breast milk
B. 4 hours for formula
C. 5 hours for breast and formula
D. 6 hours for solids
E. 8 hours for solids and 4 hours for all fluids
B. 4 hours for formula - true: "For healthy infants under 6 weeks of age having an elective procedure, formula or breast milk may be given up to four hours and clear fluids up to two hours prior to anaesthesia." ANZCA PS 15 4.5.2.3
A 12-year-old boy with spastic cerebral palsy and painful muscle spasms presents for multilevel
osteotomies of his legs. The most effective option for post-operative analgesia would be

A. a combination of NSAIDs (non-steroidal anti-inflammatory drugs) and regular tramadol
B. continuous lumbar epidural analgesia with a mixture of local anaesthetic and clonidine
C. regular paracetamol and oxycodone
D. patient controlled morphine
E. spinal morphine
B
An otherwise healthy 4-year-old presenting for adenotonsillectomy develops a cough
and laryngospasm during gaseous induction. At intubation clear secretions are visible at the
glottis and after intubation transient wheezing is noticed. Clear fluid is aspirated from
the endotracheal tube. At the end of the case, the child has a SpO2 of 96% with an
FiO2 of 0.3. Airway pressures are normal. The most appropriate further management
of this suspected aspiration is to

A. administer broad spectrum antibiotics and otherwise manage routinely
B. administer steroids and extubate after 1 hour if chest is clear on auscultation
C. extubate and observe in recovery for 4-6 hours
D. measure pH of tracheal aspirate and base further management on findings
E. perform on-table chest X-ray and base further management on findings
E. perform on-table chest X-ray and base further management on findings - the exam answer: "A chest radiograph should always be obtained if aspiration is suspected." (Kluger et al, Crisis management during anaesthesia: regurgitation, vomiting, and aspiration, in Qual Saf Health Care 2005;14:e4
Stellate ganglion is where:
A. at the level of the body of C6 (spine of C6)
B. posterior to the brachial plexus sheath
C. anterior to the dome of the pleura
D. anterior to the thoracic duct
E. anterior to scalenius anterior
?E
The best predictor of poor outcome for a peri-operative ulnar nerve injury is

A. a delay in symptom onset to more than 48 hours postoperatively
B. association with anaesthesia lasting more than 2 hours
C. association with a brachial plexus block
D. presence of bilateral injury
E. presence of mixed sensory and motor deficit
E. presence of mixed sensory and motor deficit - true: "Patients with only sensory deficits 3 months after their procedures had a greater chance of complete recovery at 1 yr than did patients with mixed sensory and motor deficits." (Warner et al, "Ulnar Neuropathy" in Anesthesiology, 1994, 81:1332-1340)
The median nerve

A. can be blocked at the elbow immediately lateral to the brachial artery
B. can be blocked at the wrist between palmaris longus and flexor carpi ulnaris
C. can be blocked at the wrist medial to flexor carpi ulnaris
D. is formed from the lateral, medial, and posterior cords of the brachial plexus
E. provides sensation to the radial half of the palm
A. can be blocked at the elbow immediately lateral to the brachial artery - false: lies medial
B. can be blocked at the wrist between palmaris longus and flexor carpi ulnaris - false: lies between FCR and PL
C. can be blocked at the wrist medial to flexor carpi ulnaris - false
D. is formed from the lateral, medial, and posterior cords of the brachial plexus - false
E. provides sensation to the radial half of the palm - true
What do C6/7 motor function do
A. flex/extension of fingers
B. flex /extend wrist
C. shoulder ext rotation / abduction
D. elbow pronation/supination
E. flexion at elbow
A c7,8
B true Last p 16
C c5
D Pronation = c7,8 Supination = c5
E c5,c6
At the wrist joint
A. the median nerve lies between the tendons of palmaris longus and flexor carpi radialis
B. the median nerve lies medial to the tendon of flexor carpi radialis
C. the radial artery is usually palpable just medial to the tendon of flexor carpi radialis
D. the ulnar artery and nerve enter the hand by passing deep to the flexor retinaculum
A. the median nerve lies between the tendons of palmaris longus and flexor carpi radialis - true: The median nerve lies superficially "with the tendon of FCR lying laterally and those of FDS and palmaris longus medially" (Anatomy for Anaesthetists 7ED p. 180)
B. the median n. lies medial to the tendon of FCU - false: See fig 124; FCU lies medial to the median nerve
C. the radial a. is usually palpable just medial to the tendon of FCU: false: radial artery is on the lateral side of the wrist
D. the ulnar artery and nerve enter the hand by passing deep to the flexor retinaculum - partly true: "The median nerve passes into the hand through the carpal tunnel" (Anatomy for anaesthetists) but the ulnar artery passes superficially.
Pre-ganglionic sympathetic fibres pass to the

A. otic ganglion
B. carotid body
C. ciliary ganglion
D. coeliac ganglion
E. all of the above
D
The peak effect of oral midazolam as a premedication in children occurs after

A. 10 - 15 minutes
B. 20 - 30 minutes
C. 35 - 45 minutes
D. 50 - 60 minutes
E. 65 - 75 minutes
B. 20 - 30 minutes - true: "The dose of oral midazolam... usually results in a satisfactorily sedated child in approximately 10-15 min with a peak effect occurring at approximately 20-30 min, with minimal to no delay in recovery, even for brief procedures." (Cote, Preoperative preparation and premedication, BJA 1999, 83:16-28)
A six-week-old baby is booked for elective right inguinal hernia repair.
An appropriate fasting time is
A. 2 hours for breast milk
B. 4 hours for formula
C. 5 hours for breast and formula
D. 6 hours for solids
E. 8 hours for solids and 4 hours for all fluids
B. 4 hours for formula - true: "For healthy infants under 6 weeks of age having an elective procedure, formula or breast milk may be given up to four hours and clear fluids up to two hours prior to anaesthesia." ANZCA PS 15 4.5.2.3
A 12-year-old boy with spastic cerebral palsy and painful muscle spasms presents for multilevel
osteotomies of his legs. The most effective option for post-operative analgesia would be

A. a combination of NSAIDs (non-steroidal anti-inflammatory drugs) and regular tramadol
B. continuous lumbar epidural analgesia with a mixture of local anaesthetic and clonidine
C. regular paracetamol and oxycodone
D. patient controlled morphine
E. spinal morphine
B
An otherwise healthy 4-year-old presenting for adenotonsillectomy develops a cough
and laryngospasm during gaseous induction. At intubation clear secretions are visible at the
glottis and after intubation transient wheezing is noticed. Clear fluid is aspirated from
the endotracheal tube. At the end of the case, the child has a SpO2 of 96% with an
FiO2 of 0.3. Airway pressures are normal. The most appropriate further management
of this suspected aspiration is to

A. administer broad spectrum antibiotics and otherwise manage routinely
B. administer steroids and extubate after 1 hour if chest is clear on auscultation
C. extubate and observe in recovery for 4-6 hours
D. measure pH of tracheal aspirate and base further management on findings
E. perform on-table chest X-ray and base further management on findings
E. perform on-table chest X-ray and base further management on findings - the exam answer: "A chest radiograph should always be obtained if aspiration is suspected." (Kluger et al, Crisis management during anaesthesia: regurgitation, vomiting, and aspiration, in Qual Saf Health Care 2005;14:e4
A 25kg child is having liver resection and is rapidly given 1 unit of blood.
Her CVP is 8. The cause most likely to be responsible for any haemodynamic
instability she experiences is:
A. coagulopathy
B. hyperkalaemia
C. ABO incompatability
D. Hypocalcaemia
E. Hypothermia
D
5yo 35kg child having repair of leg laceration. gas induction with sevo N2O
and oxygen. Can't get in drip. Put in LMA and immediately get stridor and airway
obstruction and desaturate to 90%. Next step after increase FiO2 to 100% is:
A Remove LMA and deepen with sevoflurane
B Leave LMA and deepen with sevoflurane
C Intralingual Suxamethonium
D IM Atropine
E IM Suxamethonium
A
The Neonatal Facial Coding Scale (NFCS) used to assess pain in neonates includes all of the following except
A. Brow bulge
B. Chin quiver
C. Closed mouth
D. Deep nasolabial fold
E. Eyes squeezed shut
C

Neonatal Facial Coding System (NFCS) actions monitored:

(1) brow lowering (lowering and drawing together of the brow can result in brow

bulge)

(2) eyes squeezed shut
(3) deepening of the naso-labial furrow (fold)
(4) open lips (any separation of the lips is an occurrence)
(5) vertical mouth stretch
(6) horizontal mouth stretch
(7) taut tongue (cupping of the tongue)
(8) chin quiver (high frequency vibration of the chin and lower jaw)
(9) lip pursing (tightening the muscles around the lips to form an "oo")
A young woman has an MVA when driving home from a nightclub. She was known to have
moderate MDMA (ecstasy) consumption. Which of the following complications is least
likely to be due to MDMA?
A. Hyponatraemia
B. Hypotension
C. Pneumothorax
D. Sweating
E. Tachycardia
B
The signs of exposure to a nerve agent such as Sarin or VX include

A. bronchodilation
B. dry skin
C. muscle fasciculation
D. pupillary dilatation
E. tetany
C

"The main effect of nerve agents is inhibition of acetylcholinesterase (AChE) and butaryl cholinesterase in the cholinergic nervous system. Both of these enzyme systems are familiar to anesthesiologists, who inhibit them on a daily basis using the carbamate anticholinesterase neostigmine to reverse the action of nondepolarizing neuromuscular blocking agents... The classic signs and symptoms of nerve agent poisoning are shown in Table 74-2 . Poisoning is caused by the accumulation of acetylcholine and not by the OP itself. As a result of stimulation of muscarinic synapses, there is miosis; ciliary body spasm causing pain[32]; glandular hypersecretion, including salivary, bronchial, and lacrimal; sweating; cardiac effects, including bradycardia, atrioventricular block, and Q–T prolongation; bronchoconstriction; vomiting; severe diarrhea; and fecal incontinence. The nicotinic effects are manifested by fasciculation and paralysis at the neuromuscular junction. Sometimes anomalous tachycardia may occur after nicotinic ganglionic stimulation in the sympathetic system. Central effects lead to apprehension, dizziness, amnesia, seizures, coma, and respiratory depression
A 30-year-old man presents to the Emergency Department following a high speed motor vehicle
accident. He has marked abdominal distenson, a pulse rate of 130 and a blood pressure of
80/50 mmHg. The most appropriate initial investigation would be

A. abdominal angiogram
B. abdominal paracentesis
C. CAT scan of the abdomen
D. plain X-ray of the abdomen
E. FAST (focussed abdominal sonography for trauma) scan
E
Focused Assessment with Sonography for Trauma
A young boy sustains a head injury and CT scan shows diffuse axonal injury. He is intubated
and ventilated in the ICU and is going to theatre due to rising ICP’s (20 to 30) despite
mannitol and cooling. The best anaesthetic to give is:
A. propofol and fentanyl
B. sevoflurane and remifentanil
C. isoflurane and remifentanil
D. option including nitrous oxide
E. propofol and remifentanil
E
A 30-year-old man presents to the emergency department following a high speed motor vehicle accident. His blood pressure is 70/50 mmHg with a strongly positive
FAST (focused abdominal sonography for trauma). His chest X-¬ray shows a widened mediastinum. The most appropriate method to assess the widened mediastinum in
this patient is
A. aortogram
B. CT angiogram of the chest
C. repeat chest X-ray
D. intraoperative TOE (transoesophageal echocardiography)
E. transthoracic echocardiography
D. Intraop TOE - probably the answer after much pondering: this guy needs to go theatre pronto. An on table TOE will not delay this.
The action of which drug is unchanged in a recipient following cardiac transplantation
A. Adenosine
B. Adrenaline
C. Atropine
D. Digoxin
E. Isoprenaline
E
The correct ranking of fat/blood partition co-efficients, in order of increasing solubility
in fat, for sevoflurane (S), isoflurane (I), desflurane (D) and nitrous oxide (N2O) is
A. N2O < D < S approximately = I
B. N2O approximately = D < S < I
C. N2O < S < D < I
D. D < N2O < S < I
E. D < N2O approximately = S < I
A. N2O < D < S approximately = I - true: see below

Nitrous 2.3 Desflurane 27.2 Isoflurane 44.9 Sevoflurane 47.5
The local anaesthetic LEAST likely to cause cardiac toxicity after inadvertent
intravenous injection is

A. bupivacaine
B. etidocaine
C. levobupivacaine
D. lignocaine
E. ropivacaine
D
Norpethidine toxicity
A. Is only seen if renal function is abnormal
B. Develops because the half life of norpethidine is twice that of pethidine
C. Is not seen unless treatment is prolonged
D. only seen if dose of pethidine in excess of 1.2g/day
E. May manifest early as anxiety and mood changes
E

A. Is only seen if renal function is abnormal - false
B. Develops because the half life of norpethidine is twice that of pethidine - maybe: "Although the elimination half-life of pethidine varies from 3 to 6 h, that of norpethidine is around 17 h in healthy patients, but it may be much longer if renal function is compromised."
C. Is not seen unless treatment is prolonged - false: "Numerous cases of seizures have been reported in patients receiving pethidine over a period of days or in some cases only hours."
D. only seen if dose of pethidine in excess of 1.2g/day - false
E. May manifest early as anxiety and mood changes - true: "toxic side effects such as seizures, agitation, irritability, tremors, twitches and myoclonus"
In patients with renal failure each of the following drugs has prolonged
clearance or has active metabolites with prolonged clearance except:
A. Aspirin
B. Buprenorphine
C. Codeine
D. Pethidine
E. Tramadol
B. Buprenorphine - true:

"Mean terminal half-lives are 24 hours following sublingual administration and 2 to 3 hours after parenteral injection; two-thirds of the drug is excreted unchanged, mainly in faeces, while the remaining one third is metabolised predominantly in the liver and gut wall via glucuronidation to an inactive metabolite, buprenorphine-3-glucuronide, and via CYP3A4 to norbuprenorphine, which has 40 times less analgesic effect than buprenorphine (Kress, 2009).
Which statement regarding the use of opiates for the management of acute pain is true?

A. in adults weight is the best predictor of opioid requirements
B. metabolism to codeine-6-glucuronide produces the analgesic effect of codeine
C. morphine produces more nausea and vomiting than pethidine
D. pethidine is superior to morphine in the management of renal colic pain
E. tramadol has a lower risk of respiratory depression than other opioids at equianalgesic
doses
E. tramadol has a lower risk of respiratory depression than other opioids at equianalgesic doses - true: "Tramadol has a lower risk of respiratory depression and impairs gastrointestinal motor function less than other opioids at equianalgesic doses (U) (Level II)." ANZCA pai
Plasma clearance of non-depolarising muscle relaxants in pregnant patients
(when compared with matched non-pregnant controls) is:

A. reduced because the distribution half-life is prolonged due to the increased circulating
blood volume in late pregnancy
B. reduced because the elimination half-life is prolonged due to delayed hepatic & renal
clearance brought about by hormonal changes in pregnancy
C. increased because the distribution half-life is shortened due to changes in
cardiac output in pregnancy
D. increased because the elimination half-life is shortened due to increased
hepatic and renal clearance brought about by hormonal changes in pregnancy
E. increased because the distribution half-life is shortened due to the placental
transfer of the relaxant to fetal & placental tissue.
D. Accelerated because the elimination half-life is shortened due to increased hepatic and renal clearance due to hormonal changes in pregnancy - true: "The clearance of vecuronium may be accelerated during late pregnancy, possibly relecting stimulation of hepatic microsomal enzymes by progesterone as well as by cardiovascular changes and fluid shifts that occur during pregnancy" (Stoelting Pharm and phys p.238 4th ed)
A ‘sleep dose’ of thiopentone in a healthy adult is likely to result in:
A. A fall in peripheral resistance followed by a rise in cardiac output
B. A fall in cardiac output followed by a rise in peripheral resistance
C. A fall in peripheral vascular resistance followed by a fall in cardiac output
D. A rise in peripheral vascular resistance followed by a fall in cardiac output
E. A fall in cardiac output followed by a fall in peripheral vascular resistance
C
To normalise platelet function prior to surgery, chronic diclofenac therapy
should be ceased for at least

A. 12 hours
B. 1-2 days
C. 4 days
D. 7 days
E. 10 days
A. would seem appropriate for rapidly absorbed formulations
B. would seem appropriate for slow-release formulations
Features of paracetamol administration in children include

A. limitation of the daily dose to a maximum of 150 mg.kg-1 because of the risk of hepatotoxicity
B. reliable absorption when administered rectally with most patients achieving a therapeutic
concentration with a loading dose of 20 mg.kg-1
C. peak blood levels being reached approximately 1 hour following rectal administration
D. a one hour delay between peak plasma concentration and maximum analgesia
E. a faster absorption of high dose rectal paracetamol compared to oral administration
D. a one hour delay between peak plasma concentration and maximum analgesia - true: "paracetamol should be administered 1–2 h before the desired peak effect because of a delay of 1 h between peak plasma concentrations and maximum analgesia obtained." (Howell, What we should know about paracetamol, Pediatric Anesthesia, 1999, Volume 9 Issue 4, Pages 367 - 370)
When using NSAIDs and COX-2 inhibitors for postop analgesia
A. COX-2 inhibitors are more effective analgesics than NSAIDs
B. Cox-2 inhibitors have less effect on renal function than NSAIDs
C. Cox-2 inhibitors impair platelet function
D. COX-2 inhibitors trigger aspirin induced respiratory disease with a similar
likelihood to NSAIDS
E. NSAIDs increase the risk of peri-operative bleeding after some types of surgery
A. COX-2 inhibitors are more effective analgesics than NSAIDs - false: "Non-selective NSAIDs and coxibs are effective analgesics of similar efficacy for acute pain"
B. Cox-2 inhibitors have less effect on renal function than NSAIDs - false: "Coxibs and non-selective NSAIDs have similar adverse effects on renal function"
C. Cox-2 inhibitors impair platelet function - false: "Coxibs do not impair platelet function; this leads to reduced perioperative blood loss in comparison with non-selective NSAIDs"
D. COX-2 inhibitors trigger aspirin induced respiratory disease with a similar likelihood to NSAIDS - false: "Coxibs do not appear to produce bronchospasm in individuals known to have aspirinexacerbated respiratory disease" (Acute pain management:scientific evidence p.xxii)
E. NSAIDs increase the risk of peri-operative bleeding after some types of surgery - true: "Perioperative non-selective NSAIDs increase the risk of severe bleeding after a variety of other operations compared with placebo"
A 25-year-old male presents for ECT (electroconvulsive shock therapy) at a free-standing facility.
He has a life-threatening depressive illness that has not responded adequately to medication,
however he is still taking tranylcypramine (Parnate). You should

A. cancel the procedure, cease tranylcypramine and perform the ECT in 2 weeks
B. proceed with the ECT, but induce with midazolam and remifentanil
C. proceed with the ECT, but pre-treat with esmolol
D. proceed with the ECT with caution, but with your usual drugs
E. transfer the patient to a tertiary centre for their ECT
?D
Fondaparinux Sodium (Arixtra)

A. activates platelet
B. cross reacts with sera from patients with heparin induced thrombocytopaenia
C. has a mechanism of action that is antithrombin (ATIII) dependent
D. is associated with thrombocytopaenia
E. can be safely used in patients with severe renal failure
C. has a mechanism of action that is antithrombin (ATIII) dependent - true: "Fondaparinux is a synthetic and specific inhibitor of activated factor X (Xa) with no animal sourced components. The antithrombotic activity of fondaparinux is the result of antithrombin III (ATIII) mediated selective inhibition of factor Xa. By binding selectively to ATIII, fondaparinux potentiates (about 300 times) the innate neutralisation of factor Xa by antithrombin. Neutralisation of factor Xa interrupts the blood coagulation cascade and inhibits both thrombin formation and thrombus development."
Intra-ocular pressure is increased by
A. head-up
B. hypothermia
C. metabolic acidosis
D. miosis
E. respiratory acidosis
E
In a trial, 75 patients with an uncommon, newly described complication and 50 matched
patients without this complication are selected for comparison of their exposure to a
new drug. The results show

Complication present Complication absent
Exposed to new drug 50 25
NOT exposed 25 25

From these data

A. the relative risk of this complication with drug exposure CANNOT be determined
B. the odds ratio of this complication with drug exposure CANNOT be determined
C. the relative risk of this complication with drug exposure is 2
D. the odds ratio of this complication with drug exposure is 1.33 (recurring)
E. none of the above
A. the relative risk of this complication with drug exposure CANNOT be determined - true:

"Because accurate information concerning all patients at risk in a retrospective case-control study is not available (because sample size is set by the researcher), incidence rate and risk cannot be accurately determined, and the odds ratio is used as the estimate of the risk ratio" (Myles and gin p.74)
Forty patients are randomly dived into two groups - one to receive induction agent A and another
to receive induction agent B. The next day they are asked to rate their anaesthetic experience on
a scale of 1 (very bad) to 5 (very good). The most appropriate test to compare the anaesthetic
experience of the two groups is the
A. unpaired t-test
B. Mann-Whitney test
C. Chi-square test
D. Kruskal-Wallis test
E. paired t-test
B. Mann-Whitney test - true: The scale is ordinal. "If there is a natural order among categories, so that there is a relative value among them... then the data can be considered ordinal data... Ordinal data are... a type of categorical data." (Myles and Gin p 2-3). There are two independent groups and the equivalent of the unpaired t-test for non-parametric data is the Mann-Whitney U test. "Mann-Whitney U Test (identical to the Wilcoxon rank sum) is a non-parametric equivalent to the unpaired Student's t-test" (Myles and Gin p.63) "The Mann-Whitney U test is the recommended test to use when comparing two groups that have data measured on an ordinal scale. However, if the data represent a variable that is, in effect, a continuous quantity, then a t-test may be used if the data are normally distributed. This is more likely with large samples (say n>100)." (p. 64)
A new test has been developed to diagnose a disease. To determine the SPECIFICITY of this new
test it should be administered to

A. a mixed series of patients i.e. some known to be suffering from the disease and some known
to NOT be suffering from it
B. a series of patients known to NOT be suffereing from the disease
C. a series of patients known to NOT be suffereing from the disease and an estimate of the
prevalence of the disease in the population obtained
D. a series of patients known to be suffereing from the disease
E. a series of patients known to be suffereing from the disease and an estimate of the prevalence
of the disease in the population obtained
Specificity = True Negative / True negative + False Positive Answer B sounds pretty good because if you know all the sample are negative and the test picks up that say 5% are positive, the specificity would be 95%
Correct statements regarding confidence intervals (CI) include all the following EXCEPT

A. CI are derived from the standard error (of the mean).
B. CI can be used to assess the precision of population parameter estimates.
C. The width of the CI depends on the degree of confidence required.
D. The width of the CI depends on the sample size.
E. The width of the CI depends on the mean value of the sample
E. The width of the CI depends on the mean value of the sample - false and answer to choose
When a new diagnostic test is evaluated in a population of subjects in whom the diagnosis
is known, the following results are obtained

Disease known Disease known
to be present to be absent
New test result positive 80 40
New test result negative 20 180

In this population the NEGATIVE predictive value of this test is closest to

A. 10%
B. 33%
C. 67%
D. 80%
E. 90%
E, 90%. ie TN/(TN+FN) = 180/(180+20) =180/200 = 18/20 = 9/10 = 90% NPV is the proportion of true negs out of all those testing negative, and means that the risk of someone who tests neg truly doesn't have disease is 90% in this case. Affected by prevalence, ie if disease is rare, NPV increases.